Этого треда уже нет.
Это копия, сохраненная 17 июня 2022 года.

Скачать тред: только с превью, с превью и прикрепленными файлами.
Второй вариант может долго скачиваться. Файлы будут только в живых или недавно утонувших тредах. Подробнее

Если вам полезен архив М.Двача, пожертвуйте на оплату сервера.
Тред тупых вопросов N154 679708 В конец треда | Веб
Тред вопросов о жизни, Вселенной и всём таком.

Спрашиваем то, за что в других местах выдают путёвку в биореактор. Здесь анонимные учёные мирового уровня критически рассмотрят любые гениальные идеи и нарисованные в Paint схемы.

Предыдущий тут >>672631 (OP)
moonlover 2 679715
Я изобрёл новый двигатель. Реактивный, но для межзвёздных полётов. Суть в том, что в баке и в двигателе топливо/рабочее_тело подчиняются разным физическим законам. Т.е. топливо в баке имеет меньшую массу, но попадая в двигатель оно чудесным образом увеличивает свою массу. Осталось только придумать как менять массу вещества в некотором объёме пространства.

Кстати, что такое масса? С точки зрения квантовой физики.
3 679716
>>79715
Думаю, будет достаточно разогнать двигатель до околосветовой скорости относительно бака.
4 679719
>>79716
Это СТО и очевидно что разделять двигатель и бак бесмысленно.
Нужен другой способ.

А пока следующее "изобренение" - гравитационный парус. Достаточно взять парус из веществв, которое с одной плоскости прозрачно для гравитационных волн, а с противоположной стороны не прозрачно. Складываем парус вдвое - "стоим на месте" подчиняемся законам небесной механики. Раскладываем парус и он без топлива летит куда надо.
Лучше только антигравитационный двигатель, но не факт что он возможен в принципе.
5 679724
>>79715

>подчиняются разным физическим законам.


Охуенно придумал, план надежный
image39 Кб, 921x417
6 679767
Перекатываю свой навоз с прошлого треда:

Как должна выглядеть гравитационная линза для гравитационного телескопа, чтоб она не распидорашивала объект наблюдения на 4, а фокусировала? Я так понимаю, для впуклой линзы нужен массивный тор. А что взять в качестве выпуклой, чтобы она умела в фокусировку?
image98 Кб, 400x400
8 679777
>>79772
Выглядит охуенно. Но, возможно, я ебусь в шары, но так и не понял, как они собираются реализовывать это попиксельное сканирование. Вот есть некая километровая область пространства вокруг телескопа, из каждой точки которой будет видно кольцо-пиксель, соответствующее какому-то участку наблюдаемой планеты. Но чтобы собрать в кучу всю картинку, надо будет резко успеть получить изображение кольца-пикселя с каждой точки этой километровой области, пока ничего не улетело из фокуса? Как так быстро этот телескоп будет перемещаться в пределах этой области?
9 679778
>>79777
Вероятно, "километровая" это дохуя. По идее, есть какой-то обычный сенсор обычного размера, на каждую его точку солнце фокусирует своё кольцо, получаемое с немного отличающегося угла.
10 679848
>>79772

>https://www.youtube.com/watch?v=NQFqDKRAROI


Какой смешной дядя, чтобы через 20 лет автономно сложился в цельный аппарат как лего так еще на расстоянии 650 а.е, теоретики такие оптимисты, любые инженеры поржут просто над таким проектом, тем временем Perseverance даже пробу грунта на 1.5 а.е норм собрать не может
11 679866
>>79848

>давайте придумаем тупую хуйню которая не существует ИРЛ


>ууу сила новуки

12 679874
>>79715

>Я изобрёл новый двигатель.


Зачем ты это сделал?

>Осталось только придумать как менять массу вещества в некотором объёме пространства.


Мелочи жизни. Знаешь, в 40-километровом марсианском карьере, который выкопает 1 шиз. Тебя первого и похоронят, будешь папоротник удобрять.
>>79719

>А пока следующее "изобренение" - гравитационный парус.


Еще лучше. Пчел, займись чем-нибудь более реальным. Хватит уже придумывать хренотень. И без тебя придумывателей хватает, на столетия вперёд.
>>79848
С одной стороны голос разума. С другой стороны.

>Perseverance даже пробу грунта на 1.5 а.е норм собрать не может


Вот тебе пикрил 2 вручим, и полетишь собирать, коль такой умный.
f34a5d266aa31ba7a58f9722f2cf225e.jpg20 Кб, 500x379
13 679875
>>79874

>полетишь собирать


>Имплаинг спейсачер бы отказался лететь на Марс.

14 679877
>>79875
До сих пор не понял что такое имплаинг. Но предположим да, в этом аргументе точно провал.
15 679879
>>79877
"Имплаинг спейсачер бы отказался лететь на Марс." = "Типа ты хочешь сказать спейсачер бы отказался лететь на Марс."
16 679882
>>79879
Андырстенд. Значит угадал, имплаинг это аналог "будто бы" или "как будто".
17 679899
>>79719

>Это СТО и очевидно что разделять двигатель и бак бесмысленно.


>Нужен другой способ.


>Осталось только придумать как менять массу вещества в некотором объёме пространства


Ну так СТО тебе и в помощь, просто разгоняй вещество до околосветовых скоростей и будет тебе рост массы. Правда, не совсем понятно, будет ли действительно расти тяга от этого, но тем не менее
18 679900
Какие есть варианты короткого полёта на Марс? По идее у нас сначала открывается окно перелёта Земля-Марс, потом обратно, но ко времени прилета аппарата с земли окно Марс-Земля, оказывается уже закроется. Может как-то можно через Венеру быстрее вернуться на Землю, чтоб не ждать два года на Марсе?
19 679901
>>79899
Масса при ускорении не возрастает в ТО, дегенерат. Возрастает затрачиваемая энергия.
Стикер255 Кб, 500x500
20 679904
>>79901
Энергия и есть масса.
21 679910
>>79904
Уебище тупое, энергия может переходить в массу, а масса - в энергию по эйнштейновской формуле. Однако это не означает, что энергия и масса являются одним и тем же. Например, фотон всегда обладает определенной энергией, однако его масса равна нулю по определению. Какой же ты даун.
22 679914
>>79910
Энергия твоего батхерта только что перешла в массу этого троля
fukken lold spc.jpg161 Кб, 640x400
23 679915
>>79914
Отлично сказано.
24 679917
>>79914
Очень удобно для оправдания собственной тупости. Можно спиздануть хуйню, а потом пукнуть, что это был просто "траленк".
25 679919
>>79917
Ехидный блинчик прикрепленный к картинке тебя не смутил? Кто еще тут тупой, раз такую толстоту не можешь распознать. Первый день в интернете, мальчик?
26 679930
>>79919
Тебе когда на ебальник табун негров кончает, ты им тоже говоришь, что ты их "затралел", мамин траль?
image.png302 Кб, 400x400
27 679931
>>79930
Жопу потуши уже, хватит срать.
28 679934
>>79910

>однако его масса равна нулю по определению


Давление света говорит что ты пидор
Стикер191 Кб, 500x500
29 679936
>>79934
Давление это что? Сила на единицу площади. А масса это не сила, вес это сила. А вес это не масса.
У света есть вес, но не масса, шах и мат.
30 679938
>>79934

>Масса 0


https://ru.wikipedia.org/wiki/Фотон
Швайнхунд, ты обосрался. Иди под струю мойся.
изображение.png4 Кб, 1104x33
31 679944
32 679947
Посоны, я сегодня толчке сидел и вдруг понял, что на рандомной планете может быть такое, что смена сезонов будет определяться не чредованием времён года как у нас на Земле, а состоянием между равномерным распределнием климата в одно время и жутчайший контраст льда и лавы в другое, когда планета вследствие своих вращения и прецессии, а так же из-за плолжения вокруг местной звезды и возможно влияния спутника, вдруг оказывается приливнозахваченной с точки зрения наблюдателя на поверхности, и в эти дни на терминаторе будет лютый контраст пылающих лавовых пустынь под солнцем и снежно-ледяных равнин на тёмной стороне.
Вот только сосзнать это я осознал, а вот представить условия и параметры вращения планеты в заданных рамках не смог.
Помогите составить условия, чтобы у планеты вышла такая особенность.
33 679952
>>79944

>если приписать


>если


>термин ныне выходит из употребления


Долбоеб, ты в шары долбишься или просто не читаешь свои же скрины?

Давление света происходит за счет энергии, которую несут фотоны.

>Давление электромагнитного излучения является следствием того, что оно, как и любой материальный объект, обладающий энергией E и движущийся со скоростью v, также обладает импульсом p = Ev/c². А поскольку для электромагнитного излучения v = c, то p = E/c.


https://ru.wikipedia.org/wiki/Давление_электромагнитного_излучения#Физический_смысл
Масса фотонов при этом равна нулю как и во всех остальных случаях.
изображение.png52 Кб, 2297x418
34 679958
>>79952
Энивей. У тебя с ростом скорости растет масса, законы ньютона применимы, значит выбрасывая рабочее тело на околосветовых скоростях ты получаешь не линейный рост тяги, а вот этот с корнем
35 679959
>>79947
Не-бу-дет, твердо и четко если есть атмосфера
36 679963
>>79715

>что такое масса? С точки зрения квантовой физики


Количество квантов материи.
37 679964
>>79900
Тупа перейти от дидовских реактивных движителей на технологичные безопорные - это позволит разгоняться с же пол пути и тормозить с же оставшиеся пол пути.
38 679965
>>79938
Попробуй сначала измерь массу покоя фотона.
39 679981
>>79958
При росте скорости масса не растет, швайнхунд. Это из теории относительности никак не следует. Возрастает затрачиваемая энергия, а не масса.
40 680058
>>79981
А время тоже не замедляется, просто меняется его восприятие :)
41 680126
>>80058
Истинно так!
42 680127
>>80058
Что есть время как не мера измерения скорости процессов? Взаимодействия замедляются, т.к. калибровочные бозоны проходят больший путь чем в состоянии покоя.
43 680145
>>79964
А какие варианты не из фентези?
44 680619
>>79708 (OP)
Почему на официальных фото НАСА Земля представляет собой ИДЕАЛЬНЫЙ ШАР?
45 680621
Насколько реалистично такой вид тяги?
Есть два тела. Допустим йоба массивная станция на астероиде, на ней стоит мощный ускоритель ионов, который создает узкий поток заряженных частиц. Есть другое тело, на нем стоят йоба магниты, которые отклоняют поток, попутно создавая тягу для второго объекта. Так получается отталкивающий или притягивающий луч.
46 680622
>>80619
Потому что коварные жидорептилоиды с нибиру хотят скрыть, что земля на самом деле плоская.
47 680635
>>80622
Это тупых вопросов или тупых ответов тред?
48 680636
>>80621
На втором теле должен быть источник энергии для магнитов. Либо этим потоком можно толкать второе тело только от себя, светя ему в жопу.
49 680639
Куда деваются фотоны?
Звезды выпускают фотоны, часть попадает на материю, но ведь большинство должны летать в межзвездном пространстве? Почему тогда космос черный, будет ли космос светлеть со временем, когда свободно летящих фотонов станет больше или они все поглощаются темной материей или ещё чем-то?
50 680650
>>80639
Диссипируют на эфире, пока совсем в нем не растворятся.
51 680651
>>80145
Других нет. Либо делать сказку былью, либо еле-еле летать на пердячей тяге как дряхлые диды.
52 680668
платинаЧому пусковую петлю не пилют? Типа пишут, не нужно столько грузооборота щас. Но ведь в астероидах столько митола самородного и всякого, что на триллионы лет всем хватит, захвативший их задоминирует, на лицо присядет всем. Есть же прямой стратегический интерес.
53 680671
>>80650
Из-за твоего поста захотелось чаю с зефирками.
54 680688
>>80668
Это теоретический концепт, для реализации которого нужно изобретать и отрабатывать новые технологии. Чтобы изобретать технологии тебе нужно вложить гигатонны бачей на всяких моченных без шанса вернуть бабло в перспективе 20-30 лет. Только всякие сверхгиганты промышленности или крупные государства могут позволить такие долгоиграющие инвестиции. Но у них свои приоритеты по трату бабла. Запилить автономную производственную базу на Луне уже возможно при текущих технологиях, но как там продвигается лунные программы? Сколько там бюджета освоено на отправку мясных мешков на ебучий булыжник? Больше расходов на нигеров пинающих мячик?
Ну будем честны, сейчас большинство людей похуй на всякие там космосы, им больше подавай развлечения, выяснения отношений с собой и внезапно тленность своей тушки. Научный прогресс движется в строну удовлетворения спроса большинства и не видать тебе космических мегапроектов.
55 680713
>>80639
Они редшифтятся.
56 680720
Почему 95% людей долбоебы?
57 680724
>>80688

>20-30 лет


>сверхгиганты промышленности или крупные государства могут позволить


А ты оптимист

>базу на Луне уже возможно при текущих технологиях


А ты оптимист (2)
58 680726
>>80720
Потому что люди склонны считать долбоёбами практически всех знакомых в своём кругу общения, за исключением себя, разумеется. При константе Данбара в районе 20 соотношение 95 к 5 выглядит вполне логично.
59 680757

>константа Данбара


>эффект Даннинга-Крюгера


>амнезия Гелл-Манна


>константа Уодсворта


>закон По


Как называется синдром, при котором хочется давать имена тривиальным бытовым явлениям?
Если никак, можно я назову его своим именем? Куда подавать заявку?
60 680759
>>80757
Синдром Дауна ты тоже будешь вместо названия описывать симптомами и внешними признаками?
61 680771
>>80757
То, что эмпирические правила кажутся тебе тривиальными, когда кто-то их открыл вместо тебя — это тоже проявление эффекта Даннинга-Крюгера.
62 680772
>>80619
Потому что неровности сраной земляшки на общем фоне незаметны.
Потому что земляшка и выглядит как идеальный шар, как и другие планеты.
63 680773
>>80726

>Потому что люди склонны считать долбоёбами практически всех знакомых в своём кругу общения, за исключением себя, разумеется.


Нет, у меня в кругу общения только умные люди. А вот стоит в интернет выйти так сразу заметно какие же дауны кругом, никакой логики, никакой рациональности в отличие от меня.
image80 Кб, 736x736
64 680774
>>80773

>Нет, у меня в кругу общения только умные люди.


Жалко, что ты ни с кем не общаешься.
65 680779
>>80619
Покажи фото и результаты твоих измерений формы.
66 680902
Если на экваторе больше радиации, это значит что колонистами марса будут ниггеры?
67 680907
>>80902
Это почему это на экваторе больше радиации?
68 680909
>>80907

> Это почему это на экваторе больше радиации?


Я видел фото людей пораженных радиацией. Они чернеют как банановые шкурки и становятся ниггерами.
69 680941
>>80902
Меланин защищает от ультрафиолета, от гамма-лучей вряд ли, от заряженных частиц тем более. Но ультрафиолет на Марсе тоже представляет опасность, поэтому да, лучше посылать нигеров.

>>80907
Ну очевидно, потому что атмосферы меньше между тобой и солнышком. Но на самом деле разница небольшая, потому что в радиационный фон больший вклад вносит земля.
70 680957
Дисперсия на линзе из-за разницы скорости света для разных доме волн в апреле линзы,
А на гравитационной линзе наблюдаются продольные явления?
Понятно что преломление тут не из-за разности сред, но сама массивная область обычно чем-то наполнена, и обычно чем ближе к центру тем гуще? Или пренебрежимо?
71 680959
>>80957

>доме волн в апреле линзы


Как поэтично, лол. Запомню.
Нет, гравитационное линзирование гнет одинаковы эм волны разной длины.
Во вселенной барионное вещество либо очень разреженное, либо очень плотное. Что-то посередине слишком неоднородное и рассеивает свет так, что уже бессмысленно говорить о диспепсии в нем. Впрочем диспепсия радиоволны в межзвездной плазме вполне актуальна.
72 681117
>>80941
>>80907

>в радиационный фон больший вклад вносит земля


По сравнению с солнцем может быть, по сравнению с космосом самим по себе землей можно пренебречь
73 681118
>>80959
С диспепсией ты же рофлишь? Диспепсия это расстройство желудочного пищеварения
74 681184
Откуда в атмосфере Юпитера взялся метан? Атом углерода соединился с 4 атомами водорода? Но как?
75 681186
>>81184
Гидролиз карбида алюминия, полученного, внезапно, из углерода и алюминия при высокой температуре, например.
76 681194
Это значит что на юпитере воняет говном?
77 681196
Пиздец, литературно все газовые гиганты это газонюхи вонючие. Впитали в себя весь пердеж солнечной системы и ехидно воняют во внешней солнечной системе. Банда четырех газонюхов. А юпитер еще и срет радиацией вонюче.
78 681197
>>81194
Метан ничем не пахнет, а горит он розово-желтым пламенем, а не синим. В бытовом газе примесь этантиола, чтобы воняло, ну и из за атома серы огонь становится синим
Schematic-of-cloud-layers-in-atmospheres-ranging-from-Jupit[...].png58 Кб, 850x343
79 681204
>>81184
Сгорел в водороде. Или конвертировался в восстановительной среде из изначальных окисленного углерода в СО и СО2. Скорее второе, вот тебе картиночка как на глубине меняется равновесие
80 681211
>>81186

>Гидролиз


Так жидкой воды там нет.
81 681212
>>81204

>Сгорел в водороде


углерод не горит в водороде.
82 681213
Почему бы не начать колонизацию марса с бомбёжки жизнеспособными бактериями. Может через 100500 лет, когда таки запилят настоящую колонию, там уже будет подобие экосистемы/атмосферы.
83 681215
>>81212
А хохлы?
84 681240
>>81213
К тому времени уже человечества не будет, да и не факт что смогут эволюционировать так же как на земле, застопорятся на одном уровне и се
85 681242
>>80773
Да. Люди просто как то наплевательски относяться к обработке информации. Иногда это просто рэндом. Или какой то треш.
И ведь я стараюсь из всех сил, что уже немнгого поехал на этом, и всё равно ошибаюсь логически невероятно чясто.

Поэтому 95% дебилы. Потому что быть не дибилом сложно. мозг не даёт, это усилие.
86 681318
>>81212
запруфаешь, или нахуй пойдёшь?
87 681393
Какие крупные новости были за последние пять лет?
88 681396
>>81393
Тред с жуками исчез.
89 681397
>>81396
ну охуеть. не зря дропнул /spc/ и астрономию
90 681408
>>81397
Так и мой тред удалён, это ебич админ вконец охуел от непрерывного лизания пизды негромексе и стал бросаться на людей и их тредю. Впрочем, Меркурий-тред не сравнится с Навозным. Жуки - это легенда, мы не забудем, не простим безумца. Опустите стяги, трэд Жуков погиб....
мимо оп и верный сын Ксанфомалити
да-да, этот хуй потёр и всю годноту в венера-треде
91 681430
>>81318

>запруфаешь


Тебе это долны были рассказать в 9 классе на уроке химии, даун.
92 681581
>>81430

>уроки химии


то что ты даун это понятно давно, теперь проследуй нахуй, чепушок
93 681604
>>81408

>трэд Жуков погиб


Довольно странное склонение фамили
94 681609
Колонизация солнечной системы, возможно лишь 1 путем.

1)Колонизация луны.
2)Постройка кольца(трубы) для разгона на магнитной подушке космического корабля на луне.
3)Вывоз производственных мощностей на луну.
4)Добыча на астероидах.

Ни с Марса, ни с Земли, ни с любого другого тела в солнечной системе, невозможно с такой же эффективностью колонизировать солнечную систему.
В чём я не прав?!
95 681617
>>81609
Ты забыл про то, что мясные мешки это самое слабо звено в космосе. И для колонизации человечеству нужно либо преодолеть слабость плоти и изменить себя, перестав быть человеком в биологическом смысле, либо сделать полноценный самодостаточный ИИ, который однажды поработит человечество ради оптимизации ресурсов.
96 681620
>>81617
Цифровые процессоры в космосе слабее мясных. На жалком шаттле был трижды продублирован процессор с огромным техпроцессом, по нашим временам. Современный комп ловил бы синий экран каждую минуту за пределами околоземной орбиты
97 681626
>>81609
В том что пришел в тред тупых вопросов что-то доказывать, еще и спейсача, а не сайфача
98 681654
>>81620
Хуйню несёшь. На китайском луноходе, на вертолете-на-марсе стоят обычные процы из мобилы.
Но железки еще долго уступать будут мясным манипуляторам по универсальности+точности+миниатюрности. Вот это действительно проблема как на Земле, так и в космосе.
99 681658
>>81654

>На китайском луноходе, на вертолете-на-марсе стоят обычные процы из мобилы.


Если ты посмотришь на сводки об устройстве - там в СУ шесть процессоров с резервированием, и в них как раз ехал милспек через радхард. А снап лишь в качестве экспериментальной нагрузки в системе избежания препятствий, и он как раз кадр вполне себе дропнул в одном из испытаний, плюс про общую живучесть и деградацию от накопленной эквивалентной дозы нихуя не ясно, а есть ещё ТЗЧ, которые могут произойти, а могут и не произойти за такой мелкий срок. (в полёте-то эта хуйня отключена была)
мимо
100 681666
>>80959
Кредиты за поэзию гбоарду лол.

А ещё вопрос, почему в прозрачной среде фотоны переизлучаются в том же самом направлении а не рандомно?
Выглядит как сохранение импульса, но ведь это просто различные электроники по уровням атомов среда прыг-прыг, а "положение" электрона относительно ядра будет рандомно при этом?
Как сохраняется этот же самый вектор, должен же быть ебаный пинбол и дискотека?
101 681669
>>81666

>в прозрачной среде фотоны переизлучаются в том же самом направлении а не рандомно


А это точно? В лазере-то они переизлучаются перпендикулярно накачке.
102 681676
>>81666

>А ещё вопрос, почему в прозрачной среде фотоны переизлучаются в том же самом направлении а не рандомно?


Ты про вынужденное излучение? Там есть много объяснений от полуклассической, до хардкорной КЭДной.
Полуклассическая: Возбужденный атом представляет из себя некоторый неопределившийся электрический диполь, короче как антенна с узким резонансом. Когда рядом прилетает фотон с нужной длиной волны, то он наводит на антенне ток или выводит диполь из равновесия. Ну или по другому мимо фотон заставляет определится диполь, что в итоге ведет к излучение вторичной волны, идентичной по фазе с первичной.
>>81669
Накачка лазера создает инверсию заселенности, сами фотоны нужной волны рождается из спонтанного излучения.
103 681677
>>81654

>На китайском луноходе, на вертолете-на-марсе стоят обычные процы из мобилы


Пруфы? Пока что звучит как фантастика. На последнем марсианском ровере древнее говно из нулевых, просто потому, что реже дает сбои и заточено под космос, а новых моделей нет
image.png121 Кб, 600x600
104 681685
>>80619
Убери воду. Не можешь? А вот.
>>80720
Это нормально. Вопрос философский: что есть долбоёбъ?
>>80757
Это традиция. Окстись, твоя фамилия/имярек никому не нужны. Вот откроешь, повезет тебе, тогда и приходи.
105 681686
>>80902
Нет не значит. Меланома плоха тем, что она есть и у нигр, и ее сложнее диагностировать.
>>81213
Потому же, почему бы и не начать с терраформирования венеры.
106 681702
Почему Земля не притягивает Луну?
107 681703
>>81686

> Нет не значит. Меланома плоха тем, что она есть и у нигр, и ее сложнее диагностировать.


Так если тяжело диагностировать, считай ее и нет.

Ниггам будет куда проще построить колонию на Марсе, чем беляшам
108 681705
>>79708 (OP)
Кто что думает про теорию гравитационного коллапса Вселенной?
109 681707
>>81705
Лично мне похуй
110 681710
>>81702
Притягивает же, иначе она бы не была на орбите.
111 681712
>>81710

> Притягивает же, иначе она бы не была на орбите.


Тогда зачем она отлетает на 4 см в год от Земелюшки нашей?
112 681713
>>81712
Из-за того, что Земля вращается с большей уголовой скоростью, чем вокруг нее обращается Луна, приливная волна смещается вперёд по направлению вращения Земли, опережая Луну.
113 681714
>>81712
Да и нахуй она не нужна, пусть укатывается.
114 681715
Душновато стало
115 681719
В чем проблема задачи трех тел?, если есть скажем система с тремя звездами и просто наблюдать за их движениями и делать вывод
116 681720
>>81677

>а новых моделей нет


Новых по сравнению с BAEшным RAD750? Воз и тележка. RAD5545 от того же BAE это последователь RAD750. Серия LEON, там скоро пятые пойдут. У Freescale есть серия, у TI. Куча FPGAшек и DSP - Ramon RC64, Virtex-5QV, RTG4 и т.п. У самой насы есть программа Next Generation Space Processor и вообще целое направление HPSC, и многое другое. Как раз из-за того, что мелкосерийное производство очень затратно из-за увеличивающихся цен на маски для более тонких техпроцессов.
В Perseverance используется дохуя процессоров, каждый на своей подсистеме/инструменте. В АМС никогда не бывает один ЦПУ.

>Пруфы? Пока что звучит как фантастика.


Как бы одна из главных задач вертолёта была в том, чтобы продемонстрировать работу COTS компонент в глубоком космосе. Так что там самый обычный снапдрагон 801 с линуксом стоит.
https://rotorcraft.arc.nasa.gov/Publications/files/Balaram_AIAA2018_0023.pdf
Есть один существенный нюанс, правда: снапдрагон там используется лишь для анализа картинки с камеры и работает только при "прыжках", в остальное время выключен. А сам вертолёт работает на радхард электронике от TI и BAE, и может обойтись и без камеры, это не критичный компонент. Так что продемонстрирована лишь доставка на Марс и периодическая работа в течение менее часа суммарно. При этом в одном из полётов была аномалия - был пропущен кадр с камеры. Причину не сообщали вроде, там может быть сбой и харда, и софта, и тестирования, и логики работы, точно неизвестно.
Про китайский ровер не знаю, мне кажется аноним из жопы высосал.
мимо
117 681721
>>81719

>В чем проблема задачи трех тел?


В квадратурах не решается за исключением специальных случаев.
118 681722
>>81705
И что это за "теория"? Большое сжатие что ли? Или ты, сидя на толчке, какую-то новую хуйню наманяфантазировал, шизик?
119 681723
>>81719
В аналитическом предсказании состояния этих трёх тел в будущем. Т.е. нельзя просто взять их начальные координаты-массы-скорости, посчитать формулу и точно сказать что через два миллиона лет они будут там-то. Можно предсказать только интегрально, симулируя каждый шажок вперёд во времени (очень упрощенно говоря). Особенно к этому моменту чувствителен софт для молекулярно-биологических симуляций (там решается по сути та же задача, но для миллионов тел).

А ещё в том, что система трёх тел бесконечно чувствительна к мелким погрешностям измерений и внешним возмущениям, и её поведение можно предсказать лишь на определённое время вперёд.
https://ru.wikipedia.org/wiki/Время_Ляпунова
Это время у каждой системы и погрешностей свой, но дальше него любая малейшая неточность приводит к огромным ошибкам в предсказании, ставя по сути жёсткий лимит на возможность смотреть в будущее.
120 681724
>>81722
Всё верно! Правда называть это можно по разному.
121 681726
>>81723
Детерминисты-аутисты опять соснули лол
122 681727
>>81726
Нет, интегральное решение детерминированно, математически в нём нет неопределённостей. Просто оно чувствительно к бесконечно малым погрешностям.
123 681729
>>81720

>Так что там самый обычный снапдрагон 801 с линуксом стоит.


Обвязка с микросхемами защиты от тиристорного эффекта там есть или оно для такой нагрузки не прокатит?

>картинки с камеры


А сама камера? Вроде как в среднестатистической КМОП матрице объем логики, тот который там помимо того что в самих ячейках, соизмерим ну наверное с простеньким микроконтроллером как минимум.
124 681731
>>81729
А вольфрамом не обмазывают, или бесполезно?
125 681733
>>81731
Ты о чем?
Если что, то я об одном из видов защиты. Когда заряженная частица прилетает на какой-то кусок микросхемы с несколькими транзисторами, то там возникает что-то вроде обратимого пробоя, тиристорный эффект называется. При этом микросхема мало того что обычно перестает нормально работать, так еще и через эту закоротку может начать течь большой ток, который вскоре безвозвратно выжигает этот кусок микросхемы. И вот от такого делают специальные микросхемы, которые отлавливают скачки тока и временно отключают питание чтобы оно не погорело. И такое вроде как необходимо делать в том числе и для стойких микросхем, особенно тех которые не по технологии КНИ. И по моему было бы логично в таком случае дополнительно с "простым процем из мобилы" поставить уже более простую, но стойкую микросхему защиты от тиристорного эффекта или даже несколько таких микросхем, в зависимости от того как там сделано питание.
А на деле весь эксперимент с "простым процем из мобилы" это так, ерунда. Наверняка все параметры излучения уже давно известны, можно было повторить то же самое в лабораторных условиях, что наверняка уже делалось.
Но это я так, как диванный теоретик выражаю свое мнение.
126 681734
>>81731
На самом деле не бессмысленно.
Поток гаммы и рентгена в среднем не такой уж сильный, чтобы наводить сильные помехи. В добавок они слабо взаимодействуют с вещество.
Основная проблема это релятивистские протоны, ядра гелия и нейтроны. Первые создают при торможении ворох вторичных частиц. И если один гамма фотон чаще все пройдет насквозь без последствий, то только один протон с аналогичной энергией может вызывать цепочку помех. Нейтроны же скалывают ядра вещества и наводят радиоактивность, считай прямо повреждают материал.
Торможение заряженный частиц зависит от электронной плотности и лучше все тут годится алмаз, графит или лист тупо алюминий. От нейтронов тут либо значительный слой воды/парафинов/графит или обеденный уран.
127 681737
>>81702
Притягивает.
128 681746
>>81734
Самая высокая электронная плотность у вольфрама как раз, из недорогих и нерадиоактивных веществ, выше чем у графита. Поэтому я про него и подумал
129 681747
>>81724
Большого сжатия не будет, будет тепловая смерть, т.к. Вселенная будет вечно расширяться с ускорением.
130 681749
>>81747
Анус ставишь?
15774575369430.jpg6 Кб, 275x183
131 681754
>>81746

>из недорогих

132 681755
>>81754
А что не так? Или лучше осмием обмазывать? У него электронная плотность выше
133 681757
>>81731
В смысле от радиации? Практически бесполезно. Всё что разумно блочить, блочит и тонкий алюминиевый корпус, а от ТЗЧ тебя не защитит никакой реалистический слой анобтаниума. (и то, лучше СВМПЭ какой-нибудь использовать, а не вольфрам)
134 681770
>>81746
Вольфрам тяжелее по массе. Для торможение вторичных частиц важны еще линейные размеры защиты. Защита из вольфрама будет весить в разы больше, чем из графита, при аналогичных параметрах защиты.
135 681771
>>81747
Интересно. Откуда инфа? Насколько я знаю, на данный момент Вселенная наоборот расширяется медленнее, чем например в начале. Нам известно что она расширяется из за инерции от большого взрыва, соответственно скорость должна уменьшаться. Или я чего то не догоняю?...
136 681773
>>81770
Смотря от чего. От нейтронов точно вольфрам будет тяжелее, от гаммы намного легче, масса слоя половинного ослабления у вольфрама ниже
137 681774
>>81747
Так, теория про тепловую смерть верна только в том случае если Вселенная плоская, но где доказательства что это так?
138 681775
>>81774
Что ей мешает быть не плоской и точно так же расширяться?
139 681778
>>81771
это просто терминологический казус, расширение пространства линейное и однородное на видимой сейчас вселенной, много ниже чем на инфляции, но в связи с тем что удлинённое пространство непрерывно наращивается между не связанными гравитационно объектами получается показательна функция расширения примерно 7-8% на лярд лет, так что условный куб пространства вселенной даёт непрерывно линейный прирост, но галактики расползаются с ускорением

> инерции


не, никто в душе не ебёт природу ни первичной инфляции ни тёмной энергии
140 681779
>>81773
А че не уран? Он от гаммы и нейтронов защищает.
141 681780
>>81775
Что ей мешает быть трёхмерной? Да и как можно представить двухмерное пространство, если я правильно понимаю твою идею. Насчёт себя могу добавить что существует ведь три координаты, и это факт. (Можно сказать даже четыре, но не думаю что это здесь сильно важно)
142 681781
>>81780
Когда речь идет о "плоской", то подразумевается топология в четвертом пространственном измерении, а не в привычном трехмерном. То, что мы видим сейчас, и есть плоская, гладкая как блин. А может быть с отрицательной или положительной кривизной, тор или сфера
143 681784
>>81778
Соглашусь про природу первичной инфляции и тёмной энергии. Но при чём они тут?
Про удлинённое пространство впервые слышу. И гугл тоже. Можешь поподробнее про него?
144 681785
>>81781
А, тогда становится яснее. Спорить не буду, так как за твою тему не шарю. Но спасибо за информацию, буду копать!
145 681788
>>81784
закон хаббла, чел
146 681794
>>81749
Твоей мамаши-шлюхи, да.
147 681798
>>81771
Вселенная последние 6 млрд лет расширяется с ускорением (объем пространства растет по экспоненте), с добрым утром. Ответственна за это темная энергия, которая по плотности энергии доминирует во Вселенной (ее современный вклад в полную плотность энергии Вселенной составляет 69%). Поскольку темная энергия - это космологическая постоянная скорее всего, то Вселенная будет вечно расширяться с ускорением, т.е. все закончится тепловой смертью.
148 681800
>>81774
У тебя в башке каша, дебик. Сначала узнай, что значит, что Вселенная плоская, а потом уже что-то пукай.
149 681807
>>81798
Как квантовые теории гравитации объясняют темную энергию? Опять через внешний параметр - космологическую постоянную, которая такая как есть потому что так боженька захотел?
Че там по инфлантронному полю, это гравитационный эффект или отдельное поле?
Может быть что темная материя это гравитация "протекающая" из других вселенных?
150 681808
>>81798
Почему только последние 6 млрд лет?
151 681809
>>81798

> Вселенная будет вечно расширяться с ускорением, т.е. все закончится тепловой смертью.



А почему не этой хуйнёй:

https://en.wikipedia.org/wiki/Big_Rip

https://ru.wikipedia.org/wiki/Большой_разрыв

?
152 681810
>>81807
гиперпространства закрыли при наблюдении бабаха двух нейтронок

> Как квантовые теории гравитации объясняют темную энергию?


никак, это квазисила как гравитация

> Че там по инфлантронному полю


нихуя, да и это бы по каждому чайнику бы объявили
153 681811
>>81809
потому что это маргинальная хуйня из жопы, описанная для выдуманной её автором альтернативной вселенной с другими параметрами
154 681816
>>81807
Таблетки прими, шиз.
>>81808
Потому что 6 млрд лет назад вещество в пространстве стало разреженным настолькоиз-за расширения Вселенной, что на космологических масштабах стала доминировать темная энергия, которая в ускоренном темпе начала растягивать пространство во все стороны.
155 681817
>>81809
Потому что темная энергия не является фантомом с очень высокой вероятностью. Как я уже говорил, это почти наверняка космологическая постоянная если отталкиваться от современных данных. А плотность энергии космологической постоянной, как следует из ее названия, постоянна, она не растет со временем. Поэтому Вселенная спокойно будет ускоренно расширяться бесконечно долго без всяких разрывов.
156 681822
>>81816
>>81817
Почему инфляция остановилась? Почему потом ускорение снова ускорилось? Это не константа нифига
157 681825
>>81822
Нынешнее ускоренное расширение никак не связано со стадией инфляции ранней Вселенной, долбоеб, притом что сегодняшний процесс ускоренного расширения также можно назвать инфляцией.

>Почему инфляция остановилась?


Потому что вакуум начал осциллировать - перешел из плотного высокоэнергетичного состояния в низкоэнергетичное, "выгорел" (его энергия перешла в рождение элементарных частиц, этот момент в современной космологии и называют Большим взрывом, кстати говоря). Вакуум с большой плотностью энергии - нестабильная вещь, т.к. физическая система всегда из высокоэнергетичного состояния стремится перейти в низкоэнергетичное, избавиться от лишней энергии. Кроме того, инфляция не заканчивается никогда из-за квантовых флуктуаций, она идет вечно, постоянно плодя новые вселенные с разными внутренними параметрами.

>Это не константа нифига


Швайнхунд, почему 6 млрд лет назад началось ускоренное расширение Вселенной я написал выше.
158 681826
>>81727
Это такой сорт оф сказать, что оно как бы подчиняется правилам, только исход непредсказуем. С философской точки зрения это тоже самое, что сказать что есть Бог и он управляет всем (детерминирован), но его пути неисповедимы (результат поступка зависит от непредсказуемых малых погрешностей).
159 681834

>темная энергия


И чем это отличается от святого духа?
160 681838
>>81834
Её влияние видно.
Будет забавно если это какой то эффект мультивселенной а не материя как таковая.
Вот тёмная энергия больше похоже на святой дух и эфирообразный костыль. Померяем расстояния напрямую, узнаем.
161 681844
>>81816
Спасибо! А как тебе гипотеза большого разрыва? Там тоже утверждается что Вселенная расширяется с ускорением.
162 681849
>>81826
Это просто хаотический процесс, каких множество в природе. Как динамика газов и жидкостей, тоже трудно предсказуемая хуета (схожей природы, кстати - задача трёх тел обобщается до взаимодействия любых полей и сил). Но чисто физически в самом по себе взаимодействии трех тел нет абсолютно никаких неопределённостей, как и в поведении газов и жидкостей. Разумеется существует немало процессов, способных усилить квантовые процессы до макромасштабов, вот там уже есть вопросы.

Вообще, у многих в науке распространено мнение, что бахать всё более энергетичные частицы друг об друга и открывать новые законы - вот это и есть настоящая физика. Хотя на практике обеспечить применимость уже имеющихся методов гораздо сложней. Стандартная модель очень детальна и предсказывает поведение любой физической системы с огромной точностью, однако на практике достоверно посчитать даже поведение тела и среды в граничных случаях до сих пор не представляется возможным. Тому кто укротит турбулентность - поставят памятник не хуже чем Эйнштейну.

Алсо на заметку про детерминизм, задача трёх тел полностью обратима - и есть численные методы, которые заточены под обратимость и могут посчитать систему вперёд во времени, а потом вернуть её назад в абсолютно то же состояние что и было. Естественно, в рамках границ расхождения модели. В общем принципиально там нет никакой непредсказуемости.
163 681857
>>81844
Большой разрыв это не о том, что Вселенная расширяется с ускорением, а о том, что плотность тёмной энергии увеличивается при её расширении. Зависит это от уравнения состояния w, которое, очень грубо, примерно соответствует второй производной от расширения.

Грубо говоря:
-1/3 > w > -1 — Вселенная расширяется ускоренно, но расширенное пространство содержит в себе меньшую плотность тёмной энергии, чем изначальное. Кубометров пространства становится больше, но каждый из них вносит всё меньший и меньший вклад в растяжение.

w = -1 — Вселенная расширяется ускоренно, но каждый кубометр пространства содержит такую же плотность тёмной энергии, как и было.

w < -1 — Вселенная расширяется ускоренно и каждый кубометр содержит всё больше и больше тёмной энергии, скорость расширения очень быстро (по космическим меркам) нарастает и становится бесконечной в конечное время, что и приводит к Большому разрыву.

Хорошие новости в том, что по наблюдениям w чуточку больше -1, так что никаких разрывов не будет, наша галактика останется гравитационно связанной, пока через триллионы лет не погаснут последние звёзды.
164 681861
>>81849
Спасибо за красивый и точный ответ. Я знаю. Просто был тролленастроен, ну и хотел спровоцировать тех, кто думает, что наука кайнд оф может предсказать будущее и все можно посчитать, включая решения разума людей.
165 681862
>>81844
Читай >>81817
166 681894
Что будет если мы Землю и Венеру со всеми их текущими параметрами поменяем местами?
Теперь у нас Земля с Луной на орбите Венеры, по прежнему имея ~24 часа в сутках, тот же угол наклона и та же прецессия, а Венера же по прежнему ели вращается вокруг своей оси.
Станет ли Венера жизнепригодной? Остынет ли вообще?
Высохнут ли моря и океаны экваториальной зоны Земли и превратятся ли полюса в тропики?
167 681896
>>81861
Так отсутствие возможности посчитать не отменяет детерминированности, весьма вероятной
168 681897
>>81894
Венера остынет немного, разумеется не станет и близко пригодной, а людям придется срочно мигрировать в антарктиду
169 681900
>>81857
Очень красиво рассказываешь, премного благодарствую!
170 681926
Сколько лет назад давление на Венере было таким, что она была покрыта океанами из углекислоты, ещё не успевшей перейти в сверхкритическое состояние?
171 681929
>>81926
Никогда такого не было, у кислоты слишком низкая критическая температура (меньше 400 Кельвинов). Есть версия что на Венере не сложилось тектоники плит в этом вся хуйня. Поначалу СО2 успешно связался в горных породах, но без тектоники плит процесс ограничился тонким верхним слоем коры. Слишком много СО2 осталось, и когда Солнце за миллиард-другой разгорелось (светимость на главной последовательности не постоянна, она растёт) то начался парниковый эффект. Вода ушла в атмосферу, температура поднялась на пару сотен градусов (водяной пар тоже парниковый газ), разогрелась и кора, карбонаты разложились и всё окончательно пошло по пизде
172 681955
>>79708 (OP)
Говорят, мол до Большого взрыва не было пространства и времени. Тогда как там сумели появиться и сжаться частицы? Знаю что учёным неизвестно что было до Большого взрывы, но гипотезы то насчёт этого должны быть.
173 681969
>>81955
У тебя жуткая каша в башке. Большой взрыв - начало расширения Вселенной из горячего и плотного состояния. Однако это не самая ранняя стадия эволюции Вселенной. До горячего Большого взрыва была стадия инфляции, когда пустая Вселенная экспоненциально расширялась, будучи заполненной вакуумом с огромной плотностью энергии. В конце инфляции этот вакуум начал осциллировать - его энергия перешла в рождение элементарных частиц, Вселенная заполнилась горячим плотным веществом, произошел Большой взрыв.

До стадии инфляции была Планковская эпоха, когда плотность энергии Вселенной была планковской, возраст Вселенной равнялся планковскому времени, ее размер - планковской длине, а все фундаментальные взаимодействия были слеплены в одно. А вот пройти в момент "до" Планковской эпохи уже невозможно, т.к. самого этого момента нет, у тебя нет пространства-времени за Планковской эпохой. Ты упрешься в сингулярность, т.е. кривизна Вселенной будет равна бесконечности, что означает, что число измерений и размер Вселенной равны нулю. Так что само пространство-время началось, когда началась Вселенная потому что Вселенная - это и есть пространство-время, в котором мы обитаем.
174 681982
>>81969
Откуда появилась энергия?
175 681989
>>81982
Какая энергия?
176 681991
>>81989
Которую я вложил в фонд прожиточного минимума
177 681997
Целесообразно использовать большие магнитные контуры от солнечного ветра и прочей радиации?
178 682000
>>81969

>А вот пройти в момент "до" Планковской эпохи уже невозможно, т.к. самого этого момента нет, у тебя нет пространства-времени за Планковской эпохой. Ты упрешься в сингулярность, т.е. кривизна Вселенной будет равна бесконечности, что означает, что число измерений и размер Вселенной равны нулю


Процесс создания такой сингулярности может быть перезапущен? Ну или, например, происходить локально в отдельно взятой чёрной дыре.

И почему вообще для понятия «до» применяют обычное время, которое указывает лишь направление увеличения энтропии? Может, правильнее применять «вследствие» чего-то, приведшего к формированию этой сингулярности? Может, там до начала координат энтропия со временем не росла, а уменьшалась, поэтому всё и схлопнулось перед расширением.
179 682012
>>81969
Ничем не отличается от

>Бог создал

180 682020
>>82012
Тут нужна модель бохсоздала.
181 682021
>>81997
От заряженных частиц в целом - да, целесообразно. Нужно дохуя электричества, что всегда является проблемой в космосе. Если магнит не сверхпроводящий, он будет греться и надо отводить тепло. Если магнит сверхпроводящий, то от него тем более надо отводить тепло.
От тяжелых заряженных частиц хуй защитишься. От гаммы и рентгена хуй защитишься.
182 682027
>>82000

>Процесс создания такой сингулярности может быть перезапущен?


В контексте целой Вселенной бессмысленно вообще говорить о "создании" сингулярности. Из этого понятия выводится только то, что Вселенной как таковой просто не существовало. Я выше писал, что бесконечная кривизна пространства-времени в сингулярности означает, что число измерений пространства-времени равно нулю. Т.е. пространства-времени просто нет. Сингулярность - это ничто. Вопрос "можно ли перезапустить процесс создания ничто?" абсолютно лишен всякого смысла.

>И почему вообще для понятия «до» применяют обычное время


Потому что это временное понятие, актуальное лишь в рамках временного измерения. Если у тебя отсутствует это измерение, то нет уже ни "до" ни "после" - нет совершенно ничего.

>приведшего к формированию этой сингулярности?


Я выше обрисовал, что такое сингулярность. Слова о "формировании" космологической сингулярности - это оксюморон. Формирование чего бы то ни было - процесс, и как любой процесс, он может происходить лишь в рамках пространства-времени. Сингулярность - это отсутствие пространства-времени. Формироваться было нечему и неоткуда.

>там до начала координат энтропия со временем не росла, а уменьшалась, поэтому всё и схлопнулось перед расширением.


Не было никакого "там", не было никакой энтропии и нечему было схлопываться. Ты можешь понять, что в отсутствие пространства-времени все эти понятия пропадают, лишаются всякого смысла? У тебя само время появилось, когда появилась Вселенная. Нету времени и никаких процессов "до" Вселенной, как нету ничего южнее Южного полюса.
>>82012

>пук среньк

183 682031
>>82027

>Сингулярность - это ничто.


Под горизонтом событий какая-то своя, особая сингулярность? Или ли может быть и чёрных дыр не существует?

>Потому что это временное понятие


Тыскозал? А я говорю, что причинно-следственное.

>Не было никакого "там", не было никакой энтропии


Ну ты же в курсе, надеюсь, что пруфов этого утверждения (как и его фальсифицируемости) тоже нет? Я тут, понимаешь, выйти за рамки пространства-времени пытаюсь, а ты вот это вот.

>нету ничего южнее Южного полюса


Как минимум есть области выше и ниже этой точки на сфере, например.
184 682064
Как изменится расстановка сил во внутренней солнечной системе, если мы вместо Марса поставим Уран или Нептун?
Почувствуют ли это Земля, Венера и Меркурий? Как это на них повлияет?
185 682069
>>82064
Лучше Сатурн.
186 682070
Насколько вообще стабильны орбиты планет солнечной системы?

Уже подсчитано, куда какая каменюка-газюка улетит?
187 682072
>>82070
На несколько миллионов лет вперёд гарантированно стабильны, а дальше хуй знает. Самый непредсказуемый это Меркурий, с небольшой вероятностью может улететь нахуй или ёбнуться куда-нибудь через миллионы лет.
188 682075
>>82069
Не лучше
189 682078
>>82070
Стабильность под вопросом, но более миллион лет не будет ничего происходить с орбитами.
Собственно миллион лет это крайний срок предсказаний орбит, так что ничего не подсчитано.

Про Меркурий уже сказано, но есть еще неопределенность с Юпитером. Его орбита не очень стабильна. В первую очень из-за приливов которые он наводит на другие планеты и внезапно Солнце, тем самым обмениваясь с ними орбитальным моментом. Эффект хоть очень слабый, но постоянный. За сотню миллионов лет вполне может вышвырнуть Марс, если Марс и Юпитер выйдут из орбитального резонанса.
Но это все хуйня. Если к Солнечной системе близко подлетит какую-нибудь объект около звездных масс, то возмущение в орбитах перетасует все планеты. Это уже не предсказуемый фактор. Блуждающие черные дыру чрезвычайно сложно заметить, даже если она в пределах световых лет.
190 682105
>>82078

>Если к Солнечной системе близко подлетит какую-нибудь объект около звездных масс, то возмущение в орбитах перетасует все планеты.


Уже пролетал, буквально вчера, причём не близко, а насквозь практически. https://en.wikipedia.org/wiki/Scholz's_Star#Solar_System_flyby
191 682108
>>82105

>0.82 light-years


Только чиркнуло облако Оорта, какой еще насквозь?
192 682109
Кто-нибудь видел фото ТКС в космосе? Сколько ищу, везде только рисунки и фото макетов.
193 682115
>>82108
Гравитационно - всё что ближе 2 световых лет это насквозь, ибо пробивает сегодняшнюю полость Хилла. Которая к тому же постоянно меняется, ибо дистанция между звёздами постоянно меняется и они иногда летают друг мимо друга. По этой же причине облако Оорта вряд ли вообще существует в реальности, особенно если такие пролёты часты. Оно до сих пор ничем не подтверждено, и если когда-то и существовало, то было рассеяно, ведь оно как раз должно пролегать на окраине гравитационной полости, а оттуда перехватить или выпнуть - раз плюнуть, это регион низкоэнергетических переходов с минимальными орбитальными скоростями. Либо же там обмен такими телами может идти.
194 682118
>>82109
А на него вообще иллюминаторы смотрящие были? Он же в попец стыковался к салютам.
ZaryafromSTS-88.jpg1,8 Мб, 1792x1984
195 682119
>>82109
close enough.png
196 682151
Если у фотонов нет массы, то почему чёрные дыры их притягивают?
197 682156
>>82075
Нет лучше! У Сатурна есть Титан. На орбите Марса растопится океан, океаническая будет планета. Метан из клатратов выйдет, атмосфера удвоится, климат будет тропический. К тому же выйдет и СО2 (там есть СО2). Достаточно будет запустить туда водоросли и через 100 лет будет 4% кислорода, достаточно чтобы дышать и при этом метан не горит, не репгирует. Это Венера+Земля, вот как круто!
198 682157
>>82156

>через 100 лет будет 4% кислорода


Ну ведь пизданёт же весь этот вышедший метан, где потом сратшипы заправлять?
199 682158
>>82151
Потому что она есть у черных дыр, прикинь? Любая масса искривляет пространство-время, и чем больше масса, тем сильнее искривление. Фотоны летят в искривленном пространстве-времени около черной дыры и попадают либо на фотонную сферу, либо в эргосферу, либо улетают за горизонт.
200 682159
>>82156
Титан не сможет удерживать даже азот, не говоря о метане, не надо манямир строить. А еще там будут мощнейшие восходящие потоки по всей планете из за испарения атмосферы
201 682173
>>82157
Нет, взрывоопасный процент кислорода около 11%, а не 4.
>>82159
Атмосфера уйдёт за нескоолько миллионов лет. А срок жизни человека несколько десятков лет. Про "потоки" ты кстати выдумал
202 682178
>>82173
Я не выдумал, я хуй к носу прикинул. У него потеря атмосферы будет чудовищной + кипение поверхности. Ты же в курсе, что восходящие потоки в верхних слоях атмосферы усиливают потерю атмосферы? Это замкнутая петля
.jpg49 Кб, 604x430
203 682188
>>82156

>4% кислорода, достаточно чтобы дышать и при этом метан не горит

204 682189
>>82156
Просто для понимания - на Титане облака и осадки из синильной кислоты и бензола. Там даже посрать не снимая свитер не выйдет, не то что дышать.
205 682190
>>82189

> Просто для понимания - на Титане облака и осадки из синильной кислоты и бензола. Там даже посрать не снимая свитер не выйдет, не то что дышать.


Не говоря уже о том, что там замёрзнуть без обогревателя встроенного в костюм на изичах.
206 682192
Абляционные теплощиты работают, отражая тепло от волны сжатия тонкой газовой прослойкой. Можно ли эту прослойку создавать, просто выдавливая газ из баллончика?
207 682196
>>82192
А как ты его так равномерно выдавишь? И как будешь ловить момент, когда надо начинать выдавливать? Твёрдый слой хорош тем, что он сам начинает работать там, где ПИЧОТ, и расходуется не больше, чем требуется.
208 682227
>>79708 (OP)
Мне нужно чтоб кто то размазал по фактам чепуху, которую я далее напишу. Уверенность в том, что это бред есть, а вот как это обосновать...
Из чего состоит пространство досконально неизвестно, но насколько я знаю оно точно имеет кварки (возможно в сингулярности они появились из за нарушения симметрии). Соответственно когда пространство - время находилось в сингулярности, то именно кварки и начали расширение. (Так как она была сильно нагрета, и движение кварков было дико быстрым)
А теперь, в наше время, расширение происходит засчёт тёмной энергии. Можно ли сказать, что малый (или не очень) процент тёмной энергии это те самые кварки, которые расширяли изначально пространство - время?
Скорее всего в этой цепочке проблема именно в том, как я понимаю поведение кварков (если они вообще были в тот момент), но хочется ответа от более прошаренного человека.
209 682231
>>82227

>cоответственно когда пространство - время находилось в сингулярности, то именно кварки и начали расширение


Нет, кварки и все что знает физика это постсингулярные события. Сингулярности может и не было вообще, как нельзя делить на ноль
210 682233
>>82231
Благодарю!
211 682236
>>82227
Шкварки с глютенами появились когда вселенная уже во всю экспоненциально расширялась.

>процент тёмной энергии это те самые кварки


В свободном виде их вроде как не наблюдали, но если бы они могли так существовать, их бы наверное заметили по сильному/слабому/электромагнитному взаимодействию с окружающей средой.
212 682245
Сколько негров нужно сжать в точку, чтобы получить миллиметровую черную дырочку?
213 682248
>>82245
Примерно 10 в 26 степени.
214 682249
>>82248
Много чёт. А если каждый нигга будет откормленным 200-килограммовым ниггером на стероидах?
215 682258
>>82249
Тогда в 10 в 25 степени.
216 682279
на каком расстоянии от гипотетического юпитера должен находиться гипотетический ганимед, который как массой марс, что у него гипотетически стабильно могла быть гипотетическая луна уровня плутона или энцелада?
217 682281
>>82279
Хуй знает
218 682306
Тупой вопрос, но не могу найти в интернете. Кто и когда впервые предположил, что Солнце - это такая же звезда как маленькие звёзды на небе ночью?
Gaze of the lord.png208 Кб, 535x713
219 682318
>>82178
Какое ещё кипение поверхности? У тебя на орбите Марса в 2 раза меньше света чем на Земле. И свой "хуй к носу" оставь себе, примерное время потери атмосферы я уже сказал, фантазии твои нахуй отправляются
>>82188
Читай лучше, я сказал "атмосфера удвоится" >>82156 а это значит что атмосферное давление вырастет до 3 атм. При 3 атм. 4% кислорода равны 12% при земном давлении. А этого как раз таки вполне достаточно, снабжение кислородом как на высоте 4 км
>>82189
>>82190
пшла вон, гнусная неграмотная челядь!
220 682319
>>82306
Джордано Бруно, до него подобные мысли высказывал Аль-Бируни.
221 682361
>>82318
А еще этого достаточно для горения кислорода в метане
222 682373
>>82361
Неа, смесь 48% N2, 48% CH4, 4%O2 не горит, ты обосрался
223 682380
>>82373
При коком довлении не горит?
224 682403
>>82380
Да ни при каком не горит, ещё вопросы?
225 682436
>>82403
Гений, как это у тебя "ни при каком не горит?" При росте давления некоторые смеси и без нагрева начинают взрываться, так что давление влияет
226 682437
>>82318
Метаном под давлением тебе дышать больно нравится?
227 682439
Мне недавно приснился сон, в котором Луна была гораздо ближе к Земле, что можно без сна в пути долететь даже, и обладает атмосферой с 0.3 Па SO2 (сколько именно Па посчитал когда проснулся)

Суть в том, что когда атмосфера возвращалась в жилой модуль после выхода на поверхность, всегда был запах SO2, но при посадке атмосфера никак не мешала, 0.3 Па идеально подходят

Вопрос, могла бы Луна удерживать такую атмосферу, если бы у нее был вулканизм, возобновляющий запас газа?
228 682440
>>82439
SO2 тяжёлый газ, могла бы удерживать десятки миллионов лет. Но SO2 разлагается под ультрафиолетом, скорость разложения весьма велика (срок - десятки лет). Сложно сказать насколько интенсивным должен быть вулканизм, пожалуй мой ответ - да, могла бы, будь она хоть на десятую часть такой же активной как Ио
229 682441
>>82440
Но у более массивной Ио нет такой атмосферы, даже не 0.01 Па. Радиационные пояса сдувают?
1632569038screenshot3.png208 Кб, 527x403
230 682444
Есть объяснение этому? Может быть какие-то фотки с другого ракурса или чьё-то мнение которое поясняет что это на самом деле?

Уфолог Скотт Уоринг обнаружил на Луне двухкилометровую металлическую конструкцию
Уфолог Скотт Уоринг поделился своей очередной находкой на поверхности Луны. Исследователь изучил видеозапись, полученную с помощью автоматической межпланетной станции NASA, отправленной к спутнику Земли, и увидел на ней большую металлическую конструкцию.

Источник: https://actualnews.org/exclusive/408929-ufolog-uoring-obnaruzhil-na-lune-dvuhkilometrovuju-metallicheskuju-konstrukciju.html
231 682445
>>82444
Больше всего похоже на артефакт обработки фотографии, но почему он там появился и на каком этапе я хз.
232 682446
>>82445
Там по ссылке и видос есть.
image196 Кб, 1051x806
233 682448
>>82444
Да нету там ничего, лол.

>>82446
Какой видос? Это же рендер из компиляции нескольких снимков, натянутых на сферу.
234 682450
>>82441
Да.
235 682452
>>82450
Кроме того, на Ио холодно, и SO2 и CO2 так сильно вымерзают, что их давление равно нулю.А других газов там и нету: кислород образуется с малой скоростью и, по всей видимости, сразу и сдувается, ведь он в 2 раза легче (молекула) чем SO2. Азота вулканы почти и не выделяют, как-то так
236 682482
237 682492
>>82482

>made with data from Lunar Reconnaissance Orbiter


LRO не умеет в тиктоки, это склейка фото.
238 682581
>>82492
Типа это просто много фотографий склеили в суперплавный видос? Сколько же их там было?
Алсо. Как так получилось что этот объект на всех фотографиях тогда был?
239 682582
>>82581
Вероятнее что снималось все это на линейку ПЗС, а потом натягивалось на глобус и анимировалось.
Ссылки на то какой именно там тип камеры поищи сам, ну или подожди, возможно кто-то еще принесет.
240 682583
как ты думаешь, анон, на какой границе по объёму/массе планеты уже/ещё может существовать жизнь чуть более сложная, чем одноклеточные? ну хотя бы на уровне растений, грибов и насековмых
241 682584
>>82583
Вниз - пока может поддерживать конвекцию ферромагнитных внутренностей (марс не смог и засох, например, Меркурий тоже вроде). Без магнитного поля атмосфера будет сильно больше страдать от солнечного ветра. Хотя частично разогрев планетам солнечной системы обеспечен не только массой при сжатии, но и содержанием радиоактивных изотопов выше среднего, и Луной, в случае Земли, так что может быть в иных случаях нужна планета сильно больше Земли

Вверх пока не превратится в газового гиганта

Думаю как-то так
242 682586
>>82584
Ты хочешь сказать, будь у нас вместо Земли с Луной, например, Плутон с Хароном, не видать нам даже бескрайних мховых равнин, полных тихиходок, тли, термитов и разумных муравьёв?
Эх, why live then?
243 682590
>>82583
Я сам фанат маленьких планет, поэтому расскажу тебе, что всё довольно-таки неплохо. Магнитное поле не нужно. Нужна только достаточная масса, чтобы удержать атмосферный газ от улетучивания по тепловому механизму (механизм Джинса). Механизм Джинса гласит, что если вторая космическая скорость для планеты не менее чем в 5-5,5 раз больше скорости движения молекул атмосферы, то атмосфера не улетучивается. Это самый простой базовый механизм, все другие (а их много) требуют ещё большей массы планете, чтобы атмосфера не рассеивалась.
Как же исключить все остальный механизмы, а оставить только диссипацию по Джинсу? Нужно убрать солнечный ветер, нужно убрать жёсткий ультрафиолет (менее 150 нм) и нужно сделать так, чтобы планету не бомбардировали огромные астероиды и в атмосфере не было лёгких газов на момент формирования (иначе улетучивающийся лёгкий водород, если его слишком много, заберёт с собой и азот. водорода должно быть мало). Что ж, все эти условия выполнимы:
- планета у звезды типа К
- планета это спутник большого газового гиганта
- орбита достаточно далеко от гиганта, но достаточно близко, чтобы оказаться внутри его магнитосферы
- планета сформировалась с очень толстой атмосферой из аммиака или без атмосферы но с коркой льда
- у гиганта мало спутников, планета доминирует своей массой как Титан среди спутников Сатурна
Всё, мой друг, ты получишь свою микропланету. Звезда типа К будет проявлять активность несколько сотен миллионов лет (300-500). В это время она будет излучать ультрафиолет, но не тот который разогреет экзосферу до улетучивания, а более длинноволновый, который разложит аммиак на азот и водород, а лёд на водород и кислород. За сотни миллионов лет весь водород улетучится, останется немного азота или немного кислорода. При определённом везении этого "немного"к концу периода активности звезды хватит как раз. Когда звезда утихнет, это будет спокойный карлик почти не излучающий ультрафиолета и производящий мало солнечного ветра. Весь ветер будет отклонён магнитосферой гиганта, собственное магнитное поле планете не нужно, а её масса может быть всего 3,5-4% от массы Земли. Вот так вот! Верхний предел кстати около 5-7 масс Земли, но если хочешь каменистую планету, а не водную, лучше ограничиться 5-ю. Это эмпирический предел, 5-7 масс, выше уже резко растёт доля суперземель с малой плотностью, что трактуют как наличие толстой водородной атмосферы
243 682590
>>82583
Я сам фанат маленьких планет, поэтому расскажу тебе, что всё довольно-таки неплохо. Магнитное поле не нужно. Нужна только достаточная масса, чтобы удержать атмосферный газ от улетучивания по тепловому механизму (механизм Джинса). Механизм Джинса гласит, что если вторая космическая скорость для планеты не менее чем в 5-5,5 раз больше скорости движения молекул атмосферы, то атмосфера не улетучивается. Это самый простой базовый механизм, все другие (а их много) требуют ещё большей массы планете, чтобы атмосфера не рассеивалась.
Как же исключить все остальный механизмы, а оставить только диссипацию по Джинсу? Нужно убрать солнечный ветер, нужно убрать жёсткий ультрафиолет (менее 150 нм) и нужно сделать так, чтобы планету не бомбардировали огромные астероиды и в атмосфере не было лёгких газов на момент формирования (иначе улетучивающийся лёгкий водород, если его слишком много, заберёт с собой и азот. водорода должно быть мало). Что ж, все эти условия выполнимы:
- планета у звезды типа К
- планета это спутник большого газового гиганта
- орбита достаточно далеко от гиганта, но достаточно близко, чтобы оказаться внутри его магнитосферы
- планета сформировалась с очень толстой атмосферой из аммиака или без атмосферы но с коркой льда
- у гиганта мало спутников, планета доминирует своей массой как Титан среди спутников Сатурна
Всё, мой друг, ты получишь свою микропланету. Звезда типа К будет проявлять активность несколько сотен миллионов лет (300-500). В это время она будет излучать ультрафиолет, но не тот который разогреет экзосферу до улетучивания, а более длинноволновый, который разложит аммиак на азот и водород, а лёд на водород и кислород. За сотни миллионов лет весь водород улетучится, останется немного азота или немного кислорода. При определённом везении этого "немного"к концу периода активности звезды хватит как раз. Когда звезда утихнет, это будет спокойный карлик почти не излучающий ультрафиолета и производящий мало солнечного ветра. Весь ветер будет отклонён магнитосферой гиганта, собственное магнитное поле планете не нужно, а её масса может быть всего 3,5-4% от массы Земли. Вот так вот! Верхний предел кстати около 5-7 масс Земли, но если хочешь каменистую планету, а не водную, лучше ограничиться 5-ю. Это эмпирический предел, 5-7 масс, выше уже резко растёт доля суперземель с малой плотностью, что трактуют как наличие толстой водородной атмосферы
16312767524220.jpg170 Кб, 620x465
244 682595
>>82590

>3,5-4% от массы Земли


у меня ощущение, что ты ноль уронил
на чём основан такой процент? каковы расчёты?
245 682632
>>82590

>Весь ветер будет отклонён магнитосферой гиганта


Угу, и радиационные пояса будут ебошить заряженными частицами так, что там можно загорать с корочкой будет. А под поясами твой спутник распидорит в кольцо.
246 682638
>>82632
У радиационных поясов есть предел по плотности потока.
Однако есть еще магнитный хвост в магнитосфере планеты гиганта. И там плотность потока частиц на порядки больше, чем в звездной ветре. Частицы рекомбинируют и становятся нейтральными, так они уже не отклоняются магнитным полем, и уже могут глубже проникают в атмосферу спутника и эффективно ее сдувать.
247 682639
>>82581

>много фотографий склеили в суперплавный видос


Если там склейка была сделана по тому же принципу, как, например, Google Earth работает, то фото с этим кратером было одно. Сначала запилили 3D объект с натянутой текстурой из склеенных в нужном порядке снимков, а потом плавно над ним пролетели камерой. Координаты места с этим кратером бы узнать, да на исходных (и желательно на нескольких других, под другим освещением, если есть) снимках посмотреть, что это там было.
248 682646
>>82586
Ну так атмосфера улетит ведь. Всегда будет термосфера, подогреваемая Солнцем. Хотя, можно представить такой объект ооооочень далеко от Солнца и с оооочень сильным парниковым эффектом (атмосфера из CO2) с высоким содержанием кислорода (продукт фотосинтеза), который будет связывать свободный водород чуть что. Тогда можно наверное иметь атмосферу и воду миллиарды лет. Я тоже почему-то представлял раньше мховые равнины на Плутоне, это нормально вообще?
249 682649
>>82638

>становятся нейтральными, так они уже не отклоняются магнитным полем, и уже могут глубже проникают в атмосферу спутника и эффективно ее сдувать


А за счёт чего происходит «сдувание», если частица более свободно пролетает мимо электронных оболочек и ядер? Неужели так заметно повышается процент ядер, распавшихся на более лёгкие при захвате нейтральной частицы?
250 682654
>>82649
Што? Процесс сдувания вообще на другом уровне происходит, просто нейтральный водород, например, сталкивается с молекулой в верхних слоях атмосферы, она получает дофига скорости и улетает безвозвратно
251 682657
>>82654
Ну и чем тогда такое столкновение «эффективнее» столкновения с протоном?
252 682660
>>82657
Протон отклоняется магнитным полем (в данном случае гиганта-хозяина) и вообще с атмосферой не сталкивается. Ты ночь не спал что ли?
253 682662
>>82595
У планеты массой 3,5% достаточно второй космической скорости, чтобы не дать азоту убежать. А если и не азоту, то углекислому газу точно. У Марса, кстати, достаточно массы чтобы держать а метан(!) который 16 а.е.м. , у азота 28 а.е.м., у углекислого газа 44
Дурачка с радиационными поясами не слушай, онпохоже намеренно всегда говорит в пику
254 682663
Ладно, на каком расстоянии от светила должна быть планета, чтобы солнечный ветер не сдувал атмосферу?
Я кажется, придумал уникальную комбинацию для создания жизни: планета, далёкая от своей звезды настолько, чтобы ветром не сдувало атмосферу; достаточно массивная; имеющая спутник/сама являющаяся спутником (в том числе, для магнитного поля и тектонического гравитационного подогрева.)
Земля, кстати, под эту комбинацию не подходит, так как слишком близко к Солнцу.
255 682669
>>82663
ля, чел, я тебе написал тонну текста, который основан на научных блять знаниях, на прочитаных статьях, это реалистичная модель. нет, ты продолжаешь высирать какие-то моховые плутоны "далеко от ветра", на которых в принципе не может быть тепло. и про магнитное поле продолжаешь. мдэээ, ну продолжай вариться в шизофантазиях, хуй я иит теперь кому-то что-то напишу
256 682688
Почему военные не построят в космосе рельсотронную пушку, слишком дорого или неэффективно?
257 682689
>>82688
Нет задач. Дорого и не эффективно.
Боеприпас сильно ограничен, а один выстрел будет пидорасить орбиту.
Какой-нибудь лазер лучше будет.
image22 Кб, 1000x666
258 682691
>>82689

>один выстрел будет пидорасить орбиту


Так можно же делать два выстрела одновременно, один по цели, а другой обратно для компенсации.
259 682692
>>82691
Тогда боеприпаса будет в два раза меньше.
Так же проблема с центровкой, при выстрели будет сообщаться значительный вращательный момент.
260 682699
>>82688

> Почему военные не построят в космосе рельсотронную пушку, слишком дорого или неэффективно?


1. Нахуя?
2. Картечью спутники сбивать эффективней.
261 682704
>>82688
Чтобы что? Для сбития есть ASAT наземного базирования. Ну и пока действует договор о неразмещении вооружений. а там видно будет
>>82699

>Картечью спутники сбивать эффективней.


Нет
262 682717
Возможно ли совместить химический и электромагнитный реактивные двигатели?
263 682718
>>82717
Ну чисто теоретически ничего не мешает доускорить реактивную струю каким-нибудь МГД приводом, только при типовом массовом расходе химического двигателя вгонять туда придётся охуевшие мегаватты, если не десятки мегаватт, чтобы в схеме был смысл. А их пока взять неоткуда ни с существующими реакторами, ни тем более с фотовольтаикой.

Разве что ставить батарею супер-ионисторов, которую реактор будет медленно заряжать, а потом на коротком рабочем ходе быстро разряжать.
Schebenka-Drobovik350a2mf110com.jpg460 Кб, 1414x2000
264 682799
>>82688

>Почему военные не построят в космосе рельсотронную пушку, слишком дорого или неэффективно?

265 682814
>>82799
Манясайфай от дебилов, понятия не имеющих об орбитальной механике.
266 682847
>>79708 (OP)
ВОПРОС ТАКОЙ.

Когда закончится этот ебаный цирк? Какого размера МКС и где она летает? Вам самим не стыдно, клоуны?
267 682849
>>82847
Говорят что к 30-ым годам отправят к "Миру", но это не точно.
Линейные размеры примерно 100 на 70 метров. Но это только радиаторы и солнечные панели. Жилые блоки меньше и в целом представляют Т-образные цилиндры.
Параметры орбиты и текущее положение можешь смотреть на различных профильных сайтах. Летает на высоте примерно 400 км.
268 682851
>>82849
Луна на каком расстоянии, напомни. Не 384 400 км? Давай-ка сравним размеры земли с мкс и якобы за луной. И мы охуеем. То ли лыжи не едут, то ли я не догоняю. То ли размер земли плавает, толи я должен верить в вашу масонскую жидорептилоидную хуцпу.
269 682852
>>82851
Ты очень сильно не догоняешь лол, ты хоть представляешь что такое угловые размеры? С пространственным мышлением совсем плохо? Покажи хоть фотографию, что тебя смущает
270 682855
>>82847
>>82851
ВЕРНУЛСЯ
@
НЕ ЗАВЕРНУЛСЯ

https://2ch.hk/spc/res/672631.html#677966 (М)
271 682857
>>82852
Какие размеры? Это тебе жид Эйнштейн рассказал? Ты пляшешь под его дудку и теперь меня с толку сбиваешь? Я вижу ОФИЦИАЛЬНЫЕ ФОТОГРАФИИ. ЛУНА НА ФОТКЕ И МКС НА ФОТКЕ. Луна и мкс получается сравнимые величины. А они сравнимые? Ответь, если луня якобы в четверть меньше ЗЕМЛИ нахуй. Хули с тобой говорить, ты на цех работаешь, твой подельник леонов убежал в америку и там подох или помогли, потому что не поделился с фсб.
46806original.png1,8 Мб, 1182x624
272 682858
>>82857
ШОК, ОБНАРУЖЕН ЧЕЛОВЕК ВЫШЕ НЕБОСКРЕБА
Так это работает?

А вообще, жирнич, проваливай с серьезной доски[7/spoiler]
273 682868
>>82847
Таблетки срочно прими, антисетипетух.
274 682870
>>681684 →
Наблюдательные данные "сказали", долбоеб.
275 682871
>>82868
При чем тут сети, это же Anonus ploskozemelius, особо замутироввнная форма анона, склонная к шизофрении и кретинизму. Ну или Anonus trollolo, славящиеся своей мимикрией под других представителей рода и даже семейства. Тут определитель нужен, я хоть и биолог, но не эксперт
276 682872
>>82858
Ты мне это не объясняй. Я половину жизни архитектором проработал, потом репинку окончил и делаю 3д модели. Что такое перспектива я знаю. Ты включи мозги попробуй мне объяснить, почему Земля на фоне МКС чуть-чуть больше, чем Земля на фоне ЛУНЫ, нахуй. МКС на высоте 400 км, Луна на расстоянии 384 400 км.
ТРИСТА ВОСЕМЬДЕСЯТ ТЫСЯЧ, алло.
sage 277 682889
Второй раз уже не смешно, долбоёб.
278 682910
Почему с ростом высоты (то есть, с подъёмом вверх от уровня моря), не наблюдается посинение и офиолетование неба?
Почему где-нибудь на Эвересте, в укромном месте, где нас никто на свете не найдёт небо не выглядит более фиолетовым?
Кстати, почему где-нибудь на том же Эвересте на вершинах гор снега и холода, если там, во-первых, куда ближе к экватору, чем где-нибудь у гор, фьёрдов и склонов у Баренцева моря, а во-вторых, там атмосфера должна быть тоньше, следовательно, меньше фильтрует солнечные лучи, следовательно, больше энергии долетает до поверхности, следовательно, должно быть жарче?
279 682911
>>82910
Атмосферное рассеяние фильтрует плюс-минус одну длину волны, которая может измениться разве что с составом воздуха, через который она проходит, а он довольно-таки постоянен до очень больших высот, где диссоциация начинается.
280 682913
>>82910
На поверхности намного теплее из-за парникового эффекта.
281 682921
>>82913
Какого парникового эффекта, дебс? На 8 км по твоему нет парникового эффекта? Это свойство всей атмосферы дурень. И без парникового эффекта у земли будет теплее, потому что температура растёт при движении вниз от тропопаузы, а тропопауза всегда на уровне 0,1 атм. На Эвересте, напомню, 0,3 атм. В любой атмосфере так, хоть из водорода, хоть из гелия
>>82911
А вот и нихуя. Атмосфера рассеивает всё сразу, но с разной степенью. Толще атмосфера - начинают сильней рассеиваться всё более длинные волны, атмосфера тонкая - хватает только на расаение коротких. Видимый диапазон начинается с фиолетового, соответственно вверху небо светится только фиолетовым, чем ниже тем больше появляется синего, потом зелёного и т.д. При большой толщине цвет сместится в жёлтый, т.к. короткие волны начнут полностью поглощаться в толще воздуха (многократно рассеиваться туда-сюда и затухать, а красный и жёлтый ещё дойдут до глаза
282 682923
>>82921

>На 8 км по твоему нет парникового эффекта?


Конечно нет. Точнее - почти полностью отсутствует. Потому что основная масса молекул, задерживающих излучение, сконцентрирована внизу. Задумайся, почему насо имеет возможность SOFIA гонять в атмосфере. Да потому что оно летает на высоте 12.5км, над которой уже нечему поглощать ИК излучение.

Парниковый эффект это вообще явление самых-самых нижних слоёв, на Земле задерживает тепло литералли пара-тройка км от уровня моря. Выше уже намного слабее, вплоть снеговой линии.
283 682939
>>82910
Посинение есть, офиолетования нет, т.к. человеческий глаз очень слабо чувствителен к фиолетовому свету. Спектр ​с пиком в фиолетовой области будет выглядеть синим, а с пиком в зелёной области — белым.

>>82921

>Видимый диапазон начинается с фиолетового, соответственно вверху небо светится только фиолетовым


Хуйню рассказываешь, ты же сам только что написал, что рассеяние идёт по всему спектру и на твоей же картинке нарисовано, что спектр протяжённый. Небо будет светиться всеми цветами сразу, но с разной интенсивностью — фиолетовым (больше всего), слабее синим, ещё слабее зеленым, жёлтым и красным, и т.д.

Суть в том, что человеческий глаз очень неравномерно воспринимает свет, и небо, которое сильно светится фиолетовым и слабее синим, всё равно будет выглядеть синим.
284 682946
>>82910

>Почему с ростом высоты (то есть, с подъёмом вверх от уровня моря), не наблюдается посинение и офиолетование неба?


Потому что оно просто темнеет, почему оно должно цвет менять? Уже в самолете (выше Эвереста, кстати) хорошо видно почернение кверху на 2/3 наверное
285 682964
Из чего можно сделать самодельный дешевый абляционный щит для входа в атмосферу на 1-й космической скорости? Компоненты должны продаватся в ближайшем хозмаге или их можно сделать самому.
286 682966
>>82964

>абляционный


А зачем? Десяток баллонов с пеной и картонка какая-нибудь для формирования тупого конуса. Плёнка/скотч по вкусу.
287 682967
>>82964
Асбеста со стройки напиздить. Но учитывай, что это люто токсичная штука, ибо на микроуровне асбестовая пыль это такие страшные хуёвины похожие на огрызки колючей проволоки, которые пидорасят биологические ткани только в путь.
288 682971
>>82964
Дегазованный и просмолённый спецсоставом щит, набранный из дубовых плиток - самое простое. Китайцы хотели так делать в 70х, за неимением композитов (хотя потом запилили). В кустарных условиях это сделать наверно можно, но это ядовито и нужна вакуумная камера как минимум.

>>82967
Асбест на щите разлетится к хуям ведь, его в подложку щита надо. Высокотемпературный вход (больше 2000 С) это исключительно аблятивка, нужны фенолформальдегидные смолы с подобранным под профиль входа составом, в удерживающем каркасе. Средне (1200-1600 С) - йоба-хайтек материал уровня LI-900/2200, если без аблятивки. Низкотемпературное реентри на планирующем парашюте или тросе (400 С пик) - кевлар и ему подобные.
image62 Кб, 549x557
289 683004
Если размеры галактик несколько 10-ков, 100 тысяч световых лет или например туманности на несколько световых лет, почему мы видим их без "лагов", "прогрузок", искажений? Ведь допустим с одного края объекта, который ближе к нам, свет дойдет быстрее, а от другого конца только через несколько лет, тысяч лет... короче как мне видится должна быть какая-то "волна искажений", типо как при прогрузке изображений в медленном интернете.

Недавно смотрел какое-то видео, так там этот делей зафиксировали на гравитационной линзе, свет от сверхновой был повторно зафиксирован через месяц в другом месте линзы.

НАСКОЛЬКО ВООБЩЕ МОЖНО ДОВЕРЯТЬ "ГЛАЗАМ"?
290 683012
>>83004

>"волна искажений"


В редких случаях скорости относительно размеров достаточны. Хотя я ходил на курсы по астрономии, там говорили на галактиках такое есть, если они сильно наклонены к наблюдателю ребром
291 683023
>>83004
Всё прекрасно есть.

https://en.wikipedia.org/wiki/Light_echo
292 683025
>>83023
Я все понял! Красивое название.
293 683027
>>83025
И явление красивое.
294 683028
>>83023
Но ведь это не про то совсем. Это про отражённый свет, который почти одновременно со светом исходного объекта достигает наблюдателя, из-за чего кажется, что свет от исходного объекта долетел до отражателя мгновенно. Изначальный вопрос, как я понял, в искажении самого объекта из-за того, что свет от дальнего его конца запаздывает больше, чем свет от ближнего к наблюдателю.
295 683046
>>82966
Энергия импульса тела должна во что-то преобразоватся, в данном случае - в тепло. Значит через твою тормозящую йобу нужно будет пропустить мегаджоули тепла. Пена сгорит нафиг, если не тормозить неделями, что очевидно практически невозможно.

>>82971

>щит, набранный из дубовых плиток


Дуб это слишком дорого, ты бы еще красное дерево предложил. Я так понял ты имел ввиду пористую керамику?
296 683070
>>83046

>Я так понял ты имел ввиду пористую керамику?


Нет, прям конкретно просмоленный дуб, в спутниках-шпионах FSW. Какая ещё нах керамика, у китайцев в семидесятых только говно и палки были. И носки, которые они шили. Если не ошибаюсь, потом всё-таки заменили на нормальные композиты, но изначально FSW как минимум ранних серий имели теплощит из дуба. Он маленький был (15см) и нужен был чтобы прикрыть приборный отсек с плёнкой, на всё остальное было похуй. К концу активной фазы реентри аблятивная пропитка кончалась, теплощит прогорал до угольков и постепенно тончал.
297 683082
>>83070

>прям конкретно просмоленный дуб


Можно было обойтись и березой.
Интересно, не знал.
298 683087
>>83082

>Schebenka-Drobo[...].jpg



Ты второй раз эту хуйню постишь уже, ты правда считаешь, что она настолько талантливая, оригинальная и остроумная, что заслуживает этого?
299 683088
>>83087
Комикс как комикс, немного смешной. Не знаю, чего тебя так триггерит с него.
300 683089
>>83082

>Можно было обойтись и березой.


Как я понимаю там не прочность, а какие-то другие параметры подбирались, ровность, характер прогорания и т.п.
301 683090
Зачем нужна плитка, если есть ниггеры?
302 683093
>>83089
Тоже примерно такие мысли. Предполагаю в первую очередь плотность, потом пористость для пропитки, значит лучше лиственница, наконец прочность, хотя лигнин быстро выгорит и останется чисто уголь и золы, плюс смола-наполнитель. Возможно подбирали и тестировали под ацетиленовой горелкой.
1633031000431.jpg108 Кб, 898x1400
303 683114
Вы тут какие-то плитки обсуждаете.

А вот что насчёт водянки можете подумать?

Охуенная же тема. Просто хуячим проводку из трубок, делаем подсветку чтобы красиво было, и у нас есть хорошая система охлаждения. А тестировать можно на старых фуфыксах, у них сопоставимые температуры в разгоне.
304 683117
>>83114
Проточным охлаждением сопла ЖРД охлаждают. И подсветка тоже в наличии, правда не РГБ нихуя.
image26 Кб, 654x480
305 683123
>>83028
А ведь да, я же как бы о массивных источниках излучения тоже спрашивал, а не только про точечные...
Почему-то подумалось о все этой ебаной интерференции, типо волны гасятся, уравниваются со временем и это световое эхо становиться не таким явным и заметным... хуй знает короч

даунито задавший вопрос
306 683127
>>83114
А радиатор с собой возить?
307 683128
>>83123
Так я же тебе ответил, я краем уха слышал от эксперта, что на повернутых почти ребром галактиках иногда есть крошечное "запаздывание", если быстро вращаются
308 683129
>>83127

> А радиатор с собой возить?


Можно на жопу капсулы вывести, все равно там сгорит
309 683131
>>83128
Так как это выражается то?!
310 683133
>>83131
Примерно как эффекты при вращении вентиляторов на камере, обновляющейся построчно
311 683134
>>83133
Иди нахуй со совими набросами на вентилятор.
312 683140
Представим, что Солнце превратится в черную дыру.
В таком случае на каком расстоянии будет линия горизонта события? В процентах относительно расстояния от Солнца до земли.
313 683141
При постройке коллайдера были челы, которые боялись появления черной дыры или другой непонятной хуйни.

А если всё человечество захочет сотворить что-то, то сможет ли оно это сделать на практике и в теории? Если создадут коллайдер максимально возможный и т.д.
Я не про уничтожение Земли, а про что-то большее (или специфичное), что повлияет на окружающий космос.
315 683143
>>83141
Из физически представимого наиболее близкое - экспансивный супер ИИ на основе роя нанороботов, которые сжирают к хуям всю Землю, а потом стреляют своими сгустками во все известные космические объекты, стремясь зашкварить и ассимилировать всю вселенную. В наихудшем случае за несколько миллиардов лет они могли бы обосрать всю галактику.
316 683158
>>83141
Любой сложный эксперимент всегда проходит комиссию по этике и безопасности. Мирный конечно; какой-нибудь дохуя засекреченный военный проект, где литералли оружие клепают, может и не считаться ни с чем.

>При постройке коллайдера были челы, которые боялись появления черной дыры или другой непонятной хуйни.


Тащемта не просто челы, а сами мочёные. Ещё задолго до них была проверка от не связанных ни с коллайдером, ни с CERN мочёных, на безопасность экспериментов на нём. https://cds.cern.ch/record/613175/files/CERN-2003-001.pdf
И после шумихи уже другая, на этот раз мочёными из CERN, но не участвовавшими в экспериментах, которая в свою очередь была перекрестно проверена не связанными. https://doi.org/10.1088/0954-3899/35/11/115004
317 683274
Какой силы нужен магнит, чтобы он левитировал, отталкиваясь от магнитного поля Земли?

Можно ли замутить ракету на таком двигателе, способную выйти на орбиту?
318 683276
>>83274

> Какой силы нужен магнит, чтобы он левитировал, отталкиваясь от магнитного поля Земли?


Хуй знает.

> Можно ли замутить ракету на таком двигателе, способную выйти на орбиту?


Нет. Но его можно положить на счётчик, чтобы они крутился медленнее.
319 683280
>>83274

> Какой силы нужен магнит, чтобы он левитировал, отталкиваясь от магнитного поля Земли?


Никакой, поле Земли слишком слабое для того чтобы поднять любой магнит.

>Можно ли замутить ракету на таком двигателе, способную выйти на орбиту?


Соответственно нельзя. Но можно пользоваться магнитным полем Земли для поднятия орбиты (электродинамический трос), там где тяга не критична - хотя это очень медленный даже для кубсатов способ. А также можно отталкиваться от него соленоидами для ориентации спутника в пространстве - вот это уже используют повсеместно, правда не для собственно ориентации, а для разгрузки маховиков.
320 683318
>>83274

>Какой силы нужен магнит, чтобы он левитировал, отталкиваясь от магнитного поля Земли?


Итак, у нас 0.00005Тл. Для идеального угла F.ампера=B·I·L
Нам нужно уравновесить F = mg = ρSLg
Получаем для плотности меди I/S = ρg/B = ρg/B = 8933(кг/м³)х9.8(м/с2)/0.00005(кг/Ас²) = 1 750 868 000 A/м² = 1751A/мм²

Т.е. если по катушке медного провода сечением 1мм² пустить ток 1751А - такая катушка сможет левитировать в магнитном поле земли. Медный провод от такого издевательства моментально сгорит нахуй, но вроде бы коммерческие сверхпроводники держат до 3000A/мм², а экспериментальные до 40000A/мм². Так что в теории левитировать в магнитном поле земли возможно. Правда, придётся ещё часть массы на стабилизирующий маховик пустить, потому что равновесие магнитного отталкивания будет неустойчивым, особенно вдали от полюсов.
321 683319
>>83318
К сверхпроводнику тебе нужен ещё йоба-криостат и йоба-источник энергии, а ещё он перестанет сверхпроводить в чересчур сильном магнитном поле. Маховик не нужен наверно - если у тебя йоба-электромагнит, то можно просто несколько обмоток сделать и пульсировать.
322 683320
>>83276
В нормальных счётчиках есть магнитный детектор, таких хитрожопых давным-давно вычисляют на раз
323 683341
>>79899

>будет ли действительно расти тяга от этого


С чего бы? Количество молекул то же остаётся, кпд магически не увеличивается. Вот если струю разогнать до световой скорости, но как.
324 683377
>>83318
>>83319
Ну вот и все, сверхпроводник, реактор на антиматерии и можно лететь.
325 683435
Как жёстко овнить свидетелей лунного заговора? Чтобы не опускаться в нескончаемое болото споров, демагогии и логического бега с препятствиями. 1-2-3 железобетонных последовательных аргумента, которые заставят конспиролога капитулировать.

Вот, например, плоскоземельщиков я понял как контрить - предсказательная сила официальной теории. По шарообразной земле до конца этого века сделаны точные прогнозы всех солнечных затмений с указанием времени и региона. По официальной модели всегда можно предсказать карту звёздного неба. Плоская земля в этих вопросах - только пук.
326 683439
>>83435
Спросить, почему СССР молчал.
327 683440
>>83435
А зачем? Чтобы что?
328 683449
>>83435
1 аргумент бьешь в подбородок под углом, 2 аргумент бьешь по печени, 3 аргумент добиваешь ногой на земле по ебалу
329 683451
>>83435

> По шарообразной земле до конца этого века сделаны точные прогнозы всех солнечных затмений с указанием времени и региона.


А тебе в ответ: все это и по плоской земле предсказать можно. Вряд ли при этих словах ты тут же достанешь карандаш и тетрадку и начнешь рисовать формулы и таблицы чтобы отстоять честь шароебов.
Самый простой ответ: А зачем скрывать факт плоской Земли буде она таковая?
330 683452
>>83451

>все это и по плоской земле предсказать можно


Нет) У вас на ней самая примитивная геометрия тут же разоблачает плоскую землю. Раз тебе это не ясно, то сваливай с доски
331 683455
>>83435
Смотря какой подвид лунного заговора.
1. Есть тяжёлые формы - мол, ничего никуда не летало, нет никакого космоса, нет никакой луны, ракета бы разбилась об небесный купол, земля плоская, земля вогнутая, радиация поясов мгновенно убьёт всё живое и.т.п.
2. Есть лёгкие формы - аппараты летали, но никаких людей внутри не было, ибо тогдашние СЖО не вывозили (вариант: отставание от СССР не позволяло завершить пилотируемый КА первыми). Людей потом отдельно сняли в павильоне.
3. Есть особые формы - аппараты реально летали с людьми, но в процессе произошло что-то такое, из-за чего всю программу пришлось секретить и лепить для публики фейки. Или все умерли от НЁХ, или состоялся недружественный контакт с инопланетянами, или что-то подобное.

Первый случай в общем контрится доводами в пользу круглой земли и банальной инженерной возможностью построить ракету под требуемую дельту по принципу "бери больше кидай дальше". Второй случай контрится гораздо труднее, разве что шакальными фотками с чандраянов и опытом Чернобыля по хватанию больших доз без фатальных последствий. Третий не контрится ничем. Часть гипотез класса 2 и 3 могут быть правдивыми. Например, что с людьми впервые полетел только Apollo 13 (и обосрался), недопиленные корабли 69 года летели пустыми, а высадку сфабриковали, чтобы саботировать участие СССР в гонке.

>>83451

>А зачем скрывать факт плоской Земли буде она таковая?


Классика: чтобы гои не нашли дорогу в Валинор.

У плоскоземцев тоже есть лайтовые и хардовые вариации. Например, что Земля таки шар, но значительно большего размера, чем принято считать. Или что она мультимерна, и через какие-то там дырки на полюсах можно куда-то там съебать. Сюда же идеологически примыкают версии об искаженной географии, скрывающей какие-то секретные острова или континенты. Это даже частично может оказаться правдой, ибо отретушированные области на гуглкартах точно есть.

В общем же случае для заговоров есть подход: чтобы безопасно напиздюнькать по-маленькому нужно сверху напиздеть по-большому. Допустим, чиновники втихаря торгуют золотом из Форт-Нокса на чёрном рынке - форсим версию, что золото вывозят рептилоиды за небесный купол, и над конспирологами все смеются.
331 683455
>>83435
Смотря какой подвид лунного заговора.
1. Есть тяжёлые формы - мол, ничего никуда не летало, нет никакого космоса, нет никакой луны, ракета бы разбилась об небесный купол, земля плоская, земля вогнутая, радиация поясов мгновенно убьёт всё живое и.т.п.
2. Есть лёгкие формы - аппараты летали, но никаких людей внутри не было, ибо тогдашние СЖО не вывозили (вариант: отставание от СССР не позволяло завершить пилотируемый КА первыми). Людей потом отдельно сняли в павильоне.
3. Есть особые формы - аппараты реально летали с людьми, но в процессе произошло что-то такое, из-за чего всю программу пришлось секретить и лепить для публики фейки. Или все умерли от НЁХ, или состоялся недружественный контакт с инопланетянами, или что-то подобное.

Первый случай в общем контрится доводами в пользу круглой земли и банальной инженерной возможностью построить ракету под требуемую дельту по принципу "бери больше кидай дальше". Второй случай контрится гораздо труднее, разве что шакальными фотками с чандраянов и опытом Чернобыля по хватанию больших доз без фатальных последствий. Третий не контрится ничем. Часть гипотез класса 2 и 3 могут быть правдивыми. Например, что с людьми впервые полетел только Apollo 13 (и обосрался), недопиленные корабли 69 года летели пустыми, а высадку сфабриковали, чтобы саботировать участие СССР в гонке.

>>83451

>А зачем скрывать факт плоской Земли буде она таковая?


Классика: чтобы гои не нашли дорогу в Валинор.

У плоскоземцев тоже есть лайтовые и хардовые вариации. Например, что Земля таки шар, но значительно большего размера, чем принято считать. Или что она мультимерна, и через какие-то там дырки на полюсах можно куда-то там съебать. Сюда же идеологически примыкают версии об искаженной географии, скрывающей какие-то секретные острова или континенты. Это даже частично может оказаться правдой, ибо отретушированные области на гуглкартах точно есть.

В общем же случае для заговоров есть подход: чтобы безопасно напиздюнькать по-маленькому нужно сверху напиздеть по-большому. Допустим, чиновники втихаря торгуют золотом из Форт-Нокса на чёрном рынке - форсим версию, что золото вывозят рептилоиды за небесный купол, и над конспирологами все смеются.
332 683462
>>83452

>Раз тебе это не ясно, то сваливай с доски


Великолепный аргумент, надежный как атикитерский механизм. Короче ты меня не убедил, шароеб. Земля плоская покоится на трех слонах, что стоят на трех китах, что стоят на черепахе, что плывет по морю эфира среди небесных сфер.
333 683466
>>83462
Да я уже после поста понял, что ты не плоскоземельщик, а просто моделируешь ситуацию, с утра затупил
kit2.png1,3 Мб, 1124x1024
334 683489
>>83435

>Как жёстко овнить свидетелей лунного заговора? Чтобы не опускаться в нескончаемое болото споров, демагогии и логического бега с препятствиями. 1-2-3 железобетонных последовательных аргумента, которые заставят конспиролога капитулировать.



Никак. Вообще никак. Теории заговора принципиально неопровержимы, любой довод без задней мысли объявляется сфабрикованным заинтересованной стороной. Можно разбить отдельные доводы (ну типа что радиация убьёт любого, кто полетит на Луну), но нельзя опровергнуть саму идею, что полёта на Луну не было, а была инсценировка неопределённо высокого уровня правдоподобности и продуманности против попыток её разоблачить. Чандраян окажется запущен на самом пиндосами, а если не пиндосами, то следы окажутся не следы, а наёбки от наёбывательных аппаратов. Прими это (судя по вопросу, ты уже к этому близок) и займись более полезными делами. Если тебе совсем уж невмоготу спорить с конспирологами, попробуй поинтересоваться у них самих, что могло бы теоретически заставить их изменить мнение. Сорян за такой разочаровывающий ответ, я, типа, и сам тоже не рад такой ситуации.
lvzl1009.jpg52 Кб, 348x600
335 683494
Допустим на орбите висит какая-то большая военная космическая станция и наша задача уничтожить её так, чтобы не осталось никак осколков и мусора. Реально ли это? Можно ли ёбнуть ядерной бомбой по космической станции, чтобы добиться 100% испарения без остаточного космического мусора?
Как-то по другому можно испарить космическую станцию, чтобы не создать пиздец гроб гроб кладбище пидор?
336 683496
>>83494
Мб проще отключить его и оставить на орбите? Или каким-нибудь лазером повредить ключевые элементы его работы, лазером можно светить относительно издалека кстати.
337 683497
>>83496
ну вот как бы чтобы отключить - нужно приблизиться и найти выключатель. А кто знает чем по тебе ёбнут с космической станции, если приблизишься. Лазером посветить конечно можно, но вдруг на станции баки пизданутся от твоего лазера и будет взрыв?
lpi2t029.jpg89 Кб, 714x1000
338 683499
А то так получается, что если кто-то выведет на высокую орбиту военную космическую станцию, то имеет смысл её просто сделать тупо массивной как МКС и хуй кто с этой космической станцией посмеет что-то сделать. Еще и ядерный реактор на неё поставить и пизда. Тупо будут бояться это говно тронуть, так как если ударишь по этой космической станции, она так засрёт осколками орбиту, как если сдачу даст всему живому. Какая-нибудь КНДР через лет 40 так сможет сделать. Будет на высокой орбите висеть военная космическая станция Кимерсэн-70 с лазерами, ядерным реактором и пулемётом ДШК.
339 683503
>>81747
Ну почему же? Она сейчас с положительным ускорением расширяется, потом с отрицательным ускорением будет расширяться, а потом и вообще остановится, и всё полетит обратно.
340 683504
>>83494
>>83499
Сближаешься со станцией, хватаешь её клешнёй, врубаешь движки по ретроградной. Профит. Правда реактор землю засрёт, но там реакторы маленькие, так что чернобыльских масштабов не будет.
341 683505
>>83503
А с чего бы ей? Когда расширение стало почти нулевым, оно снова начало ускоряться. Думаю это произошел "пробой" где-то, и теперь расширение не остановить
342 683506
Что будет с трупом человека, если его без скафандра оставить на поверхности Луны? А на поверхности Марса? А если покойничка выкинуть на орбиту Земли?
343 683507
>>83506
Органика будет медленно разрушаться ультрафиолетом и солнечным ветром, вода сублимировать. Дальше сам представишь. Ну на марсе песочком занесет и сильно затормозит процесс
lpi2t028.jpg107 Кб, 746x1000
344 683508
>>83504
Говорю же - на космической станции стоит пулемёт ДШК. Сблизишься, он по тебе ёбнет.
Но даже если пулёмета нет, у станции есть свои двигатели. Она просто при твоём приближении начнёт вращаться так, что ты не схватишь её - для того чтобы ты мог схватить что-то клешнёй, нужно чтобы цель сидела смирно, а если она отказывается кооперироваться и делает даже мелкие манёвры, у тебя нихуя не получится.Но допустим ты зацепился и начал опускать орбиту, станция включит свои двигатели как Наука и начнёт артачиться, начнёт поднимать орбиту, начнёт вращаться эта вся сцепка, разрушаться или же сбросит тебя.
345 683510
>>83494
Ёбнуть рядом с ней тактическую ядерную бомбу с самым маленьким зарядом который только можно получить. ЭМП хватит на то чтобы пожечь всё наглухо, а кроме него поражающих факторов в вакууме нет.
346 683511
>>83499
Прекращай принимать наркотики, нездоровое это дело
347 683513
>>83508
Прекращай принимать наркотики, нездоровое это дело
348 683514
>>83451

>Вряд ли при этих словах ты тут же достанешь карандаш и тетрадку и начнешь рисовать формулы и таблицы чтобы отстоять честь шароебов.



Ну тогда можно попросить хотя бы одну программу, основанную на модели плоской земли, по которой можно предсказать затмение и движение небесных тел. В наш век уже каждый второй школьник способен на подобное.
349 683515
Если ♂black hole♂ получается в результате смерти звезды и это по сути очень маленькая хуетень с дохуя огромной массой, получается в самом самом центре черной дыры должен быть очень тяжелый маленький камешек материи? Или там бурлит суп из неведомой хуйни? Получается вся остальная материя которая туда попадает просто утрамбовывается в этот микрокамешек? А если допустим я в античернодерном нанокостюме, я смогу ебнутся головй об эту метрию?
350 683516
>>83503

>с отрицательным ускорением будет расширяться


С каким нахуй "отрицательным ускорением"? Ты долбоеб? Таблетки прими. Если темная энергия является космологической постоянной, а все имеющиеся на сегодня данные указывают именно на это, то Вселенная будет вечно расширяться с ускорением, потому что плотность темной энергии постоянна, она не поменяется никогда.
351 683518
>>83515
Нет, для нас, внешнего наблюдателя, радиус "черной дыры" БОЛЬШЕ ее радиуса Шварцшильда, т.к. из за огромной массы она останавливает время в момент достижения этого радиуса, а значит будет асимптотически к нему стремиться и никогда не достигнет. Так что для внешнего наблюдателя не то что сингулярности не существует, но еще и горизонта событий. А вот из чего она состоит в таком подвешенном состоянии это сложный вопрос. Утрамбовывается всё на "поверхности" ака наклейки
352 683520
>>83515

>получается в самом самом центре черной дыры должен быть очень тяжелый маленький камешек материи?


В голове у тебя камешек вместо мозгов, дебил ёбаный. Внутри черной дыры, в ее середине находится сингулярность - область с бесконечным искривлением пространства-времени и бесконечной плотностью вещества. В сингулярности сосредоточена вся масса черной дыры.

>Получается вся остальная материя которая туда попадает просто утрамбовывается в этот микрокамешек?


Она разрушается в сингулярности и перестает существовать. Единственная информация, которая остается от попавшего в черную дыру вещества, - это масса.

>А если допустим я в античернодерном нанокостюме, я смогу ебнутся головй об эту метрию?


Тебя распидарасит нахуй в бесконечно искривленном пространстве-времени сингулярности. Хотя еще до этого при подлете к горизонту событий тебя приливные силы растянут в очень длинную макаронину, а потом разорвут напополам если это чд звездной массы.
353 683521
>>83520
каким образом что то может перестать существовать колдуны ученые опять сказки выдумывают в говне моченые
354 683524
>>83494

>пик


Водолаз на подводной охоте подо льдом хоккейного поля.жпг
355 683529
>>83516
Щас бы использовать для доказательства своего высера гипотетическое понятие, введённое как раз для устранения необъяснимой хуйни, происходящей с реальными наблюдениями, да ещё и с такой уверенностью утверждать о неизменности плотности.

>Ты долбоеб?

356 683535
>>83529
Антисетипетух, спок.
357 683536
>>83535
Наркоман, блядь. Полным перебором пытаешься?
358 683540
>>83494
Свести с орбиты
359 683542
>>83518

> Нет, для нас, внешнего наблюдателя, радиус "черной дыры" БОЛЬШЕ ее радиуса Шварцшильда, т.к. из за огромной массы она останавливает время в момент достижения этого радиуса, а значит будет асимптотически к нему стремиться и никогда не достигнет. Так что для внешнего наблюдателя не то что сингулярности не существует, но еще и горизонта событий. А вот из чего она состоит в таком подвешенном состоянии это сложный вопрос. Утрамбовывается всё на "поверхности" ака наклейки


Как будто Библию почитал. Ну да, у науковеров догматика примерно одинакова

>Бох так пожелал и пути неисповедимы

360 683544
>>83499
Ой блять, мысль уровня "невозможно потопить авианосец".

Достаточно повредить станцию, не обязательно распидорашивать ее в труху.
361 683545
>>81857
И чем это хорошие новости?
362 683547
>>81969
Что именно в вакууме начало осциллировать? Каша из твоего черепа?
363 683548
>>83515

> Или там бурлит суп из неведомой хуйни?


Там определенно неведомая хуйня, не объяснимая известными на сегодня формулами.
По формулам там должны быть бесконечности, но у учёных есть основания полагать, что бесконечностей нигде быть не должно вообще ну прост это уже далеко не первая ситуация, когда в формулах фигурировали бесконечности, но потом удалось все объяснить без них.

Тут даже квантовую механику не могут объяснить полностью, теории нет, тупо эмпирические эксперименты.
Например, сверхпроводимость. Эффект квантовый. Его не должно быть по теории, а он есть. Математически предсказан не был.

В черной дыре же должны быть квантово-гравитационные эффекты, то есть это буквально самая неизведанная хуйня во вселенной
15994918140890s.jpg3 Кб, 200x149
364 683552
>>83536

>пук хрюк

365 683555
>>83547
Ниггерские хуи из пасти твоей мамаши прямиком тебе в жопу.
366 683558
>>83542
Додик, тебе скучно стало и лишь бы до кого доебаться? Укатывайся в зогач
367 683637
>>83545
При w = -1.5 всё разорвало бы через десяток миллиардов лет, а так и через триллион ещё что-то тлеть будет.
368 683667
>>82851
Ты просто психически больной дегенерат, такое бывает.
369 683682
>>83521
Ты сука своим человеческим мозгом не в состоянии понять Вселенную просто.
370 683683
>>83505

>А с чего бы ей?


Это как при движении вокруг массивного объекта по эллиптической орбите с эксцентриситетом, например. Только объект это центр вселенной в данном случае.
>>83514
Все языки программирования написаны на Нибиру, очевидный аргумент.
>>83516

>С каким нахуй "отрицательным ускорением"?


Вектор которого направлен против направления движения. Зачем ты такой тупой?

>темная энергия


Она сейчас с нами, в этой комнате?
371 683684
>>83518

>Так что для внешнего наблюдателя не существует горизонта событий


Это ещё почему? Граница, где свет либо улетает в дырку, либо прилетает к наблюдателю.
>>83520

>Она разрушается


За счёт чего? До какого уровня она разрушается? Кварки?
372 683686
>>83555
Тебе в этом треде лучше задавать вопросы, чем писать ответы.
373 683688
>>83548

>Его не должно быть по теории, а он есть. Математически предсказан не был


Но он логически понятен, электрончик быстрее добежит из одной до другой точки, если его не будут пихать в бока другие частицы.
374 683689
>>83637
А чем разрыв плох? Изменение - движение вперёд; а подтухание (во всех смыслах, лол) вселенной - топтание на месте.
375 683702
>>83689

>А чем разрыв плох?


Тем, что он приведёт вселенную к гравитационно несвязанному пиздецу, какие-либо изменения в котором вообще прекратятся. Но это неточно.
https://www.youtube.com/watch?v=M6rjKQ6jgAU
376 683703
>>83683
Антисетипетух, спок. Пиздуй в зог и там ниспровергай "официальную науку".
>>83684

>За счёт чего?


За счет бесконечного искривления пространства-времени.

>До какого уровня она разрушается? Кварки?


Там и кварки сплющиваются в ничто, т.к. плотность вещества в сингулярности является бесконечной, как и кривизна пространства-времени. В условиях бесконечной плотности вещества и бесконечной кривизны пространства-времени никакое вещество ни в каком виде существовать не может.
>>83686

>пук

377 683704
>>83703

>плотность вещества в сингулярности является бесконечной


Щас бы путать бесконечность и неопределённость.
378 683705
>>83684
Потому что эта граница под материальной границей коллапсара (ЧД)
379 683707
>>83703

>Антисетипетух


Это твой протыклассник?

>За счет бесконечного искривления пространства-времени


Как ты посчитал это? Математически распиши. А лучше расскажи, как называется твоя болезнь, из-за которой ты считаешь гипотезы и теории непреложными истинами и ревностно их защищаешь?

>Там и кварки сплющиваются в ничто


Так там ничего нет?

>плотность вещества в сингулярности является бесконечной, как и кривизна пространства-времени.


Ну и что это даёт? Почему бы там не мог существовать человек во всех состояниях его составляющих частиц одновременно? Там, может быть, ты и можешь своей головой потрогать камушек, прямо как в палате роддома.

>В условиях бесконечной плотности вещества


>никакое вещество ни в каком виде существовать не может.


Ась?

>>пук


Именно, твои посты - ничего не стоящий пук.
380 683710
>>83705

>материальной границей


Это что? Граница физической "поверхности" ЧД? Разве масса ЧД не сосредоточена в одной точке?
381 683716
>>83710
Для внешнего наблюдателя - нет. Сжатие до радиуса Шварцшильда невозможно за конечное время
382 683719
>>83716
Ахиллес аж об черепаху споткнулся.
383 683721
>>83716

>Для внешнего наблюдателя


Честно говоря, проебал это уточнение. С ним ок.

>>83719
384 683723
>>83707
Жирнич, пиздуй в бэ со своим тухлым траленком. Тут тебе только поссать в ротик могут.
385 683724
>>83716
Понял, сяп.
386 683725
>>83723
Не можешь ничего объяснить и несёшь чушь ты, а тролль - я, понятно.
387 683731
внутри ГАЗовых гигантов существует хоть какая то поверхность? или это просто куча облаков ГАЗА а в центре ядро болтается?
388 683734
>>83731

>или это просто куча облаков ГАЗА а в центре ядро болтается?


Это.
>>83725

>пук

389 683737
>>83716
О, получается, что можно побывать в чёрной дыре и выбраться оттуда? Раз горизонт событий меньше радиуса самой дыры.
390 683738
>>83737

>выбраться


После окончательного испарения дыры, когда вся остальная вселенная уже давно будет в состоянии тепловой смерти?
391 683740
>>83518

>для нас, внешнего наблюдателя, радиус "черной дыры" БОЛЬШЕ ее радиуса Шварцшильда


>>83738
Нет, исходя из процитированного, можно пролететь сквозь край дыры, не пересекая горизонт событий. Правда непонятно, почему он меньше самой дыры.
392 683745
>>83740
Этот дегенерат тебе хуйню галимую заливает, не слушай его. Это либо тролль, либо какой-то умственно отсталый долбоеб либо все вместе. Из-под горизонта событий черной дыры ничто не может выбраться, даже фотоны.
393 683748
>>83745

>Из-под горизонта событий черной дыры ничто не может выбраться, даже фотоны.


Ну он обратного не писал.
394 683762
>>83731
Точно неизвестно, но скорее всего, ниже атмосферы находится +-такая же смесь газов, но под огромным давлением, т.н. "металлический водород". То есть, некое твёрдое вещество, не считая ядра, там присутствует.
395 683764
>>83745

>Из-под горизонта событий черной дыры ничто не может выбраться, даже фотоны.


Вот только переносчики гравитационного взаимодействия оттуда как-то выбираются, благодаря чему внешняя Вселенная вообще гравитационно взаимодействует с ЧД.
Ах да, у нас же "искривление", и переносчиков гравитационного взаимодействия не бывает. И дальнодействий тоже не бывает. А скорость гравитации равна скорости света. Но ЭТО ДРУГОЕ, и ей почему-то похуй, что для вылезания из-под горизонта нужна скорость выше скорости света. При этом масса под горизонтом с точки зрения внешней Вселенной на диаграммах пенроуза навечно застыла во времени, но эта же масса под горизонтом преспокойно себе летает по орбитам и срёт гравитационными волнами.
Шизофизика as is, объявили модель святой и натягиваем её везде.
396 683769
>>83745
Научись читать, прежде чем меня дегенератом называть. Я вообще не писал про веселые прыжки в чд, я только написал, что чд не образуется как таковая за конечное время из за гравитационного замедления времени. Очевидно при попытке в нее упасть произойдет "синхронизация" времени
397 683774
Если свет от солнца доходит до нас 7 минут и мы наблюдаем солнце с семиминутной задержкой, значит и закат/рассвет для нас наступает на 7 минут позже чем солнце на самом деле садится за горизонт?
398 683775
>>83764
Пиздеж, вон Купер же выбрался.
1633364778369.mp4401 Кб, mp4,
640x276, 0:09
399 683776

>по математике Нобелевскую премию не вручают, потому что математика не наука, а какая-то абстрактная, не существующая в реальности хуйня


>Нобелевская премия мира: существует

400 683778
>>83764
А ещё, черные дыры как-то сливаются друг с другом, хотя сингулярность должна застыть во времени и чтобы одной сингулярности долететь до другой, нужна вечность
401 683780
402 683781
>>83764
Шиз, тебе в зог, воевать с "официальной наукой". Таблетки только перед этим принять не забудь.
>>83769

>чд не образуется как таковая за конечное время


Это и есть хуйня галимая, которую ты тут заливаешь, совершенно не разбираясь в теме, долбоебушка. Поэтому дегенератом я тебя назвал абсолютно справедливо.
403 683785
>>83781
Какой же ты дурачок, пиздец просто. Кроме говна ничего не высказал, зато самым умным остался.
kosmos-1686-1.jpg26 Кб, 400x397
404 683786
>>82109
Чел, хоть и поздно, но держи. Это фото Салюта-7 к которому пристыкован Космос-1696, один из ТКС. Фото экспедиции Союз Т-15, которая кстати еще и перелетала в космосе по маршруту Мир-Салют7-Мир.
405 683787
>>83774
Нет.
>>83780
Ты долбоёб просто. Закат солнца происходит из-за того, что Земля поворачивается и перекрывает свет, а не из-за того, что с Солнцем что-то происходит. Солнце могло быть хоть в миллиарде световых лет от нас, но само событие заката происходит от тебя на расстоянии горизонта, т.е. около 5 км, если стоять на поверхности.

Задержка между моментом, когда планета перекрывает солнечный свет и моментом, когда ты это видишь, составляет всего пару десятков микросекунд.
406 683788
>>83774
Нет, на запаздывание рассвета/заката влияет только расстояние до наблюдаемого края горизонта. Свет, долетевший до Земли за 7 минут, почти мгновенно перекрывается этим краем при повороте Земли, похуй сколько он там до этого летел.
1024px-Eratostene--CalcoloRaggioTerrestre.jpg180 Кб, 1024x1006
407 683789
Не уверен вообще, что мне помогут на спейсаче с вопросом о геофизике первого класса.

Ну вдруг. Буду рад любому объяснению без агрессии для гуманитария, а то я уже весь извелся за последние дни, пытаясь это понять.

В общем. Эратосфен еще в каком-то третьем веке до нашей эры посчитал длину окружности Земли. Вопрос не к тому как он это сделал, а к тому, что написано про это в википедии.

https://en.wikipedia.org/wiki/Eratosthenes

И вот что я никак не могу понять (а я уже даже эксперименты дома провел с пенопластовым шаром со спичками вставленными и фонариком), почему так блять важно, чтобы точки были на одном меридиане? Какая вообще нахуй разница. Мой опыт говорит мне, что как только хотя бы тень одной точки пропадает, т.е. когда фонарик в зените, то тень от второй точки автоматически всегда выстраивается по прямой, на которой лежат две точки этих воткнутых в шар спичек. Можно крутить шар как угодно, как и фонарик, но как только одна из теней пропадает, вторая автоматически будет на той же прямой, что в общем-то уже с легкостью дает нам возможность нарисовать это все на плоскости и посчитать.

Второе что я не понимаю, это какая блять разница на северном тропике там этот город блять или нет? Вот какая? уже просто того факта, что там солнце бывает в абсолютном зените должно быть достаточно. Зачем про это вообще написано на вики?

У меня абсолютно спокойно вышло посчитать окружность моего шара и без этих двух факторов.

В общем буду рад, если кто-то сумеет объяснить прямо на пальцах, ибо я очень тупой.

Говоря проще почему было не взять абсолютно два любые города на планете между которыми было известно на тот момент точное расстояние, и так чтобы один из них был тупо где-нибудь ниже северного тропика, чтоб там можно было наблюдать солнце в зените. Почему именно надо на одном меридиане и почему именно на линии тропика.
408 683793
>>83781
Пукнутый, это ты? Узнал тебя по бессодержательным постам и отсутствию понимания темы. Как ты, тёмную материю нашёл у себя в очке уже?
409 683797
>>83789
Может быть у нас разные версии статьи в вики, но из того, что прочёл я, следует, что метод Эратосфена утерян, и никаких упоминаний меридианов нет.
Нет никакой разницы, между какими точками мерять, можно даже и без солнца в зените обойтись, главное правильно углы замерить.
410 683798
>>83785
>>83793

>пук хрюк

411 683799
>>83797

>метод Эратосфена утерян


Ну, ты понял о чем я. Там есть краткая выжимка метода записанная другим ученым современником. И вот там написано, что он исходил из предположений, что они на одном мередиане и сиена на северном тропике. Но я просто не понимаю какая нахуй разница и почему это якобы важно.

>Нет никакой разницы, между какими точками мерять, можно даже и без солнца в зените обойтись, главное правильно углы замерить.



Я о том же. Правда вряд ли можно было обойтись без солнца в зените. Потому что этот фактор просто напросто позволил сделать это одному человеку, без синхронизации измерений в двух городах. Он просто слышал, что в Сиене есть такое любопытное свойство, как отсутствие теней в день солнцестояния, и это позволило ему не навещая Сиену просто замерить длину тени в тот же день в Александрии. А иначе пришлось бы побывать во-первых и там, и там, а во-вторых еще и на глаз определять кратчайшую прямую от одного города до другого.
5046086C3C0C720531.jpg104 Кб, 800x600
412 683809
>>83731

>ГАЗовых


>ГАЗА

413 683812
На одном меридиане просто полдень наступает в один и тот же момент, как ты и пишешь, не надо синхронизировать часы.
414 683815
>>83812
Это сюда >>83799
415 683824
>>83812
Храни тебя Господь. Я наконец успокоился.
416 683864
>>83781
О, маняэксперта подвезли
417 683879
>>83864

>пук

418 683929
Диаметр Млечного Пути - 100 000 св. лет. Масса черной дыры, матки - 4 300 000 солнечных масс.

Диаметр галактики Сомбреро - 50 000 св. лет. Однако масса черной дыры - 1 000 000 000 солнечных.

Как такое возможно?!?!?!
419 683936
>>83929
А ты посчитай, сколько будет весить Стрелец А*, когда всосёт Млечный путь, скукожив его до 50 000 световых лет.
420 683941
>>82946
>>82939
Так, а на какой высоте начитается видно резкое посинение-почернение с разделением неба на чёрный оксмос, белую полосочку и синюю полосу у поверхности? То есть, если бы у нас Эверест был бы не горой с пиками, а равномерным плато, забравшись на которое, то можно было бы увидеть это самое посинение неба и разделение на фракции ещё даже не на вершине Эвереста?

Кстати, а если взять и поместить на орбиту нашей Луны небесный камушек размером с пельмешку, то ничего не будет, и камушек спокойной будет вращаться на лунной орбите, и ни гравитация Земли, ни Солнца, камушек этот не сдвинет. Формально, это уже будет спутник спутника, но вопрос не в том. Если мы этот камень размером с пельмень увеличим до размера хинкали? А до размера манты? А до размера арбуза? И в какой момент такой спутник Луны окажется настолько весомым , чтобы уже Земля его начала колебать и чтобы его гравитация влияла на мимоспутники?
421 683949
>>83941

>И в какой момент такой спутник Луны окажется настолько весомым


Ну во-первых от увеличения его массы влияние на него Земли и Солнца не увеличится. Только он сам начнет более заметно влиять на Луну, и только поэтому будет колбаситься. Можешь в Universe Sandbox 2 поэкспериментировать с этой конкретной ситуацией, думаю на орбите ~150 км даже тело размером с Фобос будет стабильно от недели до года, потом Луну чиркнет неизбежно и упадет на нее

>Так, а на какой высоте начитается видно резкое посинение-почернение с разделением неба на чёрный оксмос


Думаю на километрах 15 уже будет очень ясно, что что-то здесь сильно не так. Хотя опять же, на 11 км в самолете видно, что небо затемнилось. Причем настолько, что я в самом начале сентября в ночной перелет видел в иллюминатор созвездие Ориона на самом горизонте, несмотря на стекла, пластик и блики из салона
422 684072
>>83941

>Кстати, а если взять и поместить на орбиту нашей Луны небесный камушек размером с пельмешку, то ничего не будет, и камушек спокойной будет вращаться на лунной орбите, и ни гравитация Земли, ни Солнца, камушек этот не сдвинет.



Гравитационное ускорение не зависит от массы самого тела — и ты, и муравей, и синий кит будете испытывать одинаковые ускорения.

Твой камушек будут двигать и гравитация Земли, и Солнца, и сильно неравномерная плотность самой Луны. Большая часть низких окололунных орбит люто нестабильна и спутник быстро разобьётся о Луну, как, разбился, например, PFS-2 с Аполлона-16. С начальной орбиты 130×90 км масконы его раскачали до падения на поверхность всего за 1 месяц.

Чтобы камушек пролетал хотя бы год, нужно подбирать специальную орбиту, где разные возмущения компенсируют друг друга, но даже тогда орбита будет изменяться со временем и рано или поздно твой камушек либо въедет в поверхность, либо его выкинет с орбиты Луны.
423 684157
>>83776
Прост у Нобеля математик бабу увёл в своё время.
424 684169
>>83776
В чём заключается тупой вопрос?
425 684212
>>84157

> Прост у Нобеля математик бабу увёл в своё время.


Какие же математики альфачеды
426 684260
Господа, вопрос жизни и смерти. Помню был гайд на orbiter, на русском. Нигде не могу найти.
1633463043973.png703 Кб, 720x521
427 684262
>>83787
Благодарю постера за верный ответ
428 684265
Насколько ярко светит квазар в активной галактике?

Допустим, у нас в центре галактики был бы квазар. Было бы его видно невооружённым глазом днём?
429 684271
>>84260
Поищу у себя через до конца недели, но сильно не рассчитывай.
430 684272
>>84271
Спс. Меня интересует полный гайд с сайта http://kulch.spb.ru/Doc/Tutorial_1/orbitaloperations_rus.htm но он чому-то не работает.
431 684281
>>84265
Ебанулся что ль? Квазары хуярят с космологических расстояний чуть ли не через всю видимую часть Вселенной. Если бы в нашей Галактике работал квазар, его днем на Земле было бы не просто видно - от потока квантов разных диапазонов, в т.ч. оптического, вся аппаратура захлебнулась бы нахуй.
432 684307
>>84265
Щас посчитаю. Возьмем светимость самого яркого квазара из открытых, 600 триллионов солнечных. До центра галактики 50к световых лет, а это три миллиарда с небольшим астрономических единиц. Т.е. квазар был бы со светимостью 6 х 10^-5 от солнца лол. Это будет звезда -16.2 звездной величины. Ярче полной луны во много раз, но намного тусклее солнца. Днем конечно будет видно, но это самый яркий открытый квазар. Был бы он тусклее раз в сто-тысячу, уже были бы проблемы с тем чтобы его заметить
433 684393
>>84307

>До центра галактики 50к световых лет


25к, а не 50.
434 684394
>>84393
Ну категория та же, значительно ярче полной луны и колоссально тусклее солнца
435 684407
Поиграл в StarBound, наткнулся на планету с деревьями-розами с бутонами, размером с автомобиль Ока.
Задумался.
Если бы на Земле была бы более плотная атмосфера на уровне плотности х=[1;10], где 1 = атмосфере Земли, а 10 = атмосфере Венеры, при условии что все остальные параметры атмосферы сугубо земные, и что атмосфера просто увеличивается количественно, не изменяясь в пропорции, то на что бы это повлияло? Это, я думаю, могла бы привести к плотному уровни воздуха у поверхности, к огромным деревьеподобным травам и кустам, а уж деревья, предполагаю, вообще стали бы титаническими; насекомые же имея такую плотную и атмосферу скорее всего бы и вымахали до размеров коровы или даже больше. Это понятно, это ясно. А как бы это всё сказалось на климате?
436 684415
>>84407

>х=(1;10]


Пофиксил
437 684423
>>84407
Насекомые дышат трахеями, расстояние от поверхности трахеи до тканей не должно превышать расстояние, которое слабо зависит от концентрации и давления кислорода. Вероятно, насекомые были бы крупнее, но не намного. Для пауков размером с корову нужны легкие или их аналог. Теоретически таковые вполне могли быть у одной из вымерших ветвей членистоногих.
438 684424
>>84423

>не должно превышать расстояние


>не должно превышать определенный предел


фикс
439 684430
>>84407
>>84423
Гигантские членистоногие - следствие высокого процентного содержания кислорода, а плотная атмосфера вообще как должна влиять на размер чего-либо? Это же не g.
440 684433
>>84430
Подумал, что плотная атмосфера всё-таки повлияла бы на форму существ: они бы стали более аэродинамичны, но более "энергосберегательные", слоупочные. Ну и всё-таки более мускулистые.
441 684449
>>84433

>более мускулистые


Тащемта наоборот, зачем сильно напрягаться при полёте в плотной среде. А если ещё суметь в управляемую плавучесть, можно вообще расслабиться. Алсо сравни подводных и сухопутных млекопитающих, насколько беспомощные и неповоротливые водятся хуитки в воде, как более плотной среде.
442 684451
>>84449
Да, согласен. Да и вообще энергосбережение и мускулы ниоч согласуются, мда.
443 684463
>>84430

>Гигантские членистоногие - следствие высокого процентного содержания кислорода


Почему ты думаешь что более высокое парциальное давление кислорода автоматически должна увеличить размеры насекомых? Оно ускорит диффузию кислорода из трахей в ткани, но не внутри тканей. Еще нужно помнить про необходимость вывода углекислого газа, скорость которого одинакова при любом содержании кислорода. Скорее всего это и станет ограничивающим фактором. Пауков-коров не будет (без нового типа органов дыхания, повторю) и слава Богу.
444 684465
>>84463
Как же хочеца рогатую паукорову скачущую по деревьям, размахивая паутиной из сисек.
USOEZouNpho.jpg445 Кб, 912x1111
445 684467
>>84465
Если она будет ласковой и не станет кусатся, я тоже не против.
446 684485
>>83451

>А зачем скрывать факт плоской Земли буде она таковая?


Чтобы направить нас по неверному пути. Что Бога нет, что Гитлер в Вальхалле не сидит, что нет потустороннего рая на земной тверди. Наука лишила нас надежды на лучшее будущее.
447 684495
>>83778
Хороший вопрос, поддерживаю
Мне тоже не нравится эти бесконечное искривления бесконечное еще что то, я думаю что лет через 50-100 на эти бесконечные свойства черных дыр будут смотреть как щас на эфир
448 684499
>>84495

>Мне тоже не нравится


Всем похуй, что тебе там не нравится. Наличие сингулярности внутри черной дыры математически доказано и напрямую вытекает из ОТО. Или ты шизик, который отрицает ОТО?
15105252329770.jpg5 Кб, 130x200
449 684500
>>84499

>Наличие сингулярности внутри черной дыры математически доказано и напрямую вытекает из ОТО.


Аргумент уровня "движение быстрее скорости света математически доказано и напрямую вытекает из ньютоновской механики".
450 684509
>>84499
Ты же долбоёб, который цепляется к рандомным фразам, игнорируя вполне конкретные неудобные тезисы вроде

>черные дыры как-то сливаются друг с другом, хотя сингулярность должна застыть во времени


Зачем ты нужен?
451 684512
>>84500

>пук


>>84509
Чушок, я не вижу никакого тезиса. Вижу только какой-то бессмысленный шизовысер. Что тут комментировать? На хуй иди со своей шизофазией, даун.
452 684513
>>84512
Ты же слепой долбоёб, который цепляется к рандомным фразам, игнорируя вполне конкретные неудобные тезисы вроде

>черные дыры как-то сливаются друг с другом, хотя сингулярность должна застыть во времени


Как ты это делаешь?
15723702652060s.jpg5 Кб, 259x194
453 684533
>>84513

>Ты же слепой долбоёб, который цепляется к рандомным фразам, игнорируя вполне конкретные неудобные тезисы вроде


>>черные дыры как-то сливаются друг с другом, хотя сингулярность должна застыть во времени


>Как ты это делаешь?

454 684552
>>81969

>расширялась, будучи заполненной вакуумом с огромной плотностью энергии



>пустая



что такое пустая тогда? Что там было если даже вакуума не было?
455 684607
>>84552
Не было никакого вещества, никаких частиц, имеется в виду. Были только квантовые флуктуации и высокоэнергичный вакуум.
456 684610
Cука в какое душное время живём. Нихуя красивого на небе нет.
457 684626
>>84610
А в то время, как на небе будет вот это всё, уже нихуя красивого не будет на земле.
458 684634
>>84463
Это не моё мнение, а теория (наверно, самая распространённая), выдвинутая учёными.
Вообще странно, в чём проблема вырастить несколько сотен поколений каких-нибудь мушек в герметичном контейнере с большим содержанием кислорода и проверить эту теорию.
>>84485
Как шизы будут контрить сам факт существования горизонта? >>84512
Так не комментируй, сиди молча, за умного сойдёшь.
>>84607
Что за энергия такая в вакууме? У нас реальная энергия бывает только кинетической, насколько я помню.
459 684636
>>84610
Ты се равно это не увидишь без выезда в области без засветки
460 684688
>>84634
Ненулевая плотность энергии вакуума, глупенький чушок. В модели Алексея Старобинского вакуум обретает огромную плотность энергии и отрицательное давление под действием большой кривизны пространства, дающей квантовый эффект типа эффекта Казимира.
461 684706
>>84499

> шизик, который отрицает ОТО?


Как там происходит случка ОТО с квантовой теорией поля? А с теорией инфляции / большого взрыва?

ОТО - такой же костыль, как и ньютоновская механика.
462 684708
>>83778
Придумал хуйню: если пространство-время дискретно, то бесконечной кривизны существовать не может. То есть, стенки черной дыры напоминают такую пиксельную лесенку.
А ничто не может вырваться наружу из-за того, что ячейки пространства-времени значительно меньше размеров частиц.
Тогда и сингулярности никакой нет. Внутри ЧД просто дефолтные реакции как в звезде, прост температуры в триллионы градусов
463 684709
>>84706

>А с теорией инфляции / большого взрыва?


Вся современная космология, в т.ч. теория инфляции, основывается на ОТО, лол блядь. Попизди мне еще тут хуйню.

>ОТО - такой же костыль, как и ньютоновская механика.


Ясно, антисетипетух в треде. Пиздуй в зог воевать с "официальной наукой", тут тебе делать нехуй.
Blackbody.png94 Кб, 960x768
464 684722
>>84708
Ты только что формулу Планка. Как там дела с квантуемостью гравитации?
465 684723
>>84709

>Вся современная космология


У нас вроде не sci-fi доска, чтобы фанфики обсуждать.
466 684742
>>84723
У тебя есть рецензированные научные работы по твоим фанфикам?
Наука знает что не знает всего, теории сменяют друг друга.
Так это и работает, а ты требуешь "ололо истины в последней инстанции"
Не будет её.
Вероятно, не будет никогда.
467 684764
кто может научить играть в эндлесс спейс 2?
468 684775
>>84764
Маляры в /gsg/
469 684782
Почему космическое пространство на огромных масштабах перерастает в непонятные структуры?

То есть у нас есть кусок камня в космосе и вокруг него по орбите движутся более мелкие камушки. Все это повторяется в масштабах вплоть до галактик. А дальше какие то сетевые структуры, ОСИ, СТЕНЫ, что за пиздец?
470 684783
>>84742
Я всего лишь против того, чтобы теории принимать за истину, как некоторые шизы здесь.
471 684784
>>84776 (Del)
>>84533
>>84512
>>84499
>>83798
>>83781
Вот этот понос отсюда ещё вытравить бы, вообще норм стало бы.
472 684786
Я правильно понимаю что в космосе насрать на то какое у тебя оружие, и что 22 калибр лонг райфл будет равнозначен 5.45?
473 684788
>>84786
Да. Порох не сможет воспламениться. С таким успехом можно и палкой угрожать
474 684789
>>84786
С чего это? И в каком смысле равнозначен? Останавливающее действие, летальность, отдача, всё будет разное.

>>84788
Ноуп, в патроне есть окислитель, современное оружие может стрелять и под водой, и в вакууме.
475 684790
>>84782
Вообще, вполне естественные структуры, и очень даже понятные, очень похоже на некоторые другие процессы, которые просто в быту можно наблюдать с какими-нибудь липкими субстанциями. Ну, и вообще симуляции этого процесса делали, ничего удивительного там нет:
https://www.youtube.com/watch?v=FBkYIqtYb0I
476 684792
>>84789

> С чего это? И в каком смысле равнозначен? Останавливающее действие, летальность, отдача, всё будет разное.


С того, что пулька 22 калибра в состоянии убить человека.

Если человек в скафандре, то для того чтобы продырявить скафандр большая мощь не нужна. В итоге имеем имбу в виде автомата с большим магазином и лёгким боезапасом.
477 684797
>>84790
Обычно если структуры естественные то они повторяются так или иначе. Где например увидеть повторение огромной оси?
478 684798
>>84790
И чем обусловлены стены материи и их отсутствие
479 684800
>>84792
Достаточно какой-нибудь герметик выдумать, который будет застывать в вакууме, лол.
480 684801
Как будет вести себя неньютоновская жидкость если в нее влетит объект на высокой скорости?
481 684802
>>84801
Как камень хрупкий типа гипса
482 684803
>>84802
В теории вообще возможно сделать смесь, которая будет принимать на себя удар вместо обшивки станции/корабля?
483 684804
>>84803
Нет, она же разобьется и разлетится осколками-брызгами
484 684806
>>84804
Так она в процессе затвердевания не берет на себя часть энергии?
485 684807
>>84806
Берет, но ударная вязкость низкая выходит. Она у правильным образом сплетенных волокон куда выше
486 684809
>>84784

>пук


Антисетипетуху больно.
>>84782
Так проявляются неоднородности плотности вещества во Вселенной на космологических масштабах.
487 684810
>>84797

>огромной оси?


Какой еще оси? Нет никакой "оси" блядь.
488 684812
>>84797
Осью ("ось зла") называют не физический объект, а вероятное нарушение анизотропии. Название просто показывает нежелательность такого явления для космологии, т.к. в очередной раз придется манямирок перестраивать
489 684814
А КАК ЖЕ ВРАЩЕНИЕ ОГРОМНЫХ МАСС ВОКРУГ ЦЕНТРА

Все ясно тут фанатики и веруны большого взрыва
490 684815
>>84814
Антисетипетух, спок.
491 684816
>>84800

> Достаточно какой-нибудь герметик выдумать, который будет застывать в вакууме, лол.


Он будет выдуман, а оружие 22 калибра уже сейчас существует.

Я думаю на всяких лунах и марсах будут активно юзать, так как калибры больше тупа юзлесс.
492 684822
>>84812
Нет никакой "оси зла", карта реликтового излучения гауссова, т.е. представляет собой случайный шум. Все эти "оси зла" и прочая хуйня, которую там якобы видят некоторые горе-моченые, - это то же самое, когда в облаках или в коре деревьев видят всякие страшные и не очень ёбла/силуэты/образы и т.п. Мозг работает с глюками.
https://naked--science-ru.cdn.ampproject.org/c/s/naked-science.ru/article/nakedscience/os-zla-kak-strannaya?amp

>К первому апреля 2011 года в архиве электронных препринтов вышла статья «Нестандартные космологические реликтовые паттерны в космическом микроволновом фоне» (arXiv:1103.6262), в которой авторы издеваются над поисками «паттернов» в реликтовом излучении, проверяя карту на корреляции с символами ☺, ☹, ликом Христа на Туринской плащанице и еще парой картинок. И, конечно, они находят значимую корреляцию. Чтобы не мелочиться, они оценивают значимость корреляции в ∞ σ. Этот е-принт мог бы по праву войти в сборник «Физики шутят». Кстати, там четыре автора, фамилии всех начинаются на «Z» (Zuntz, Zibin Zunkel, and Zwart) и все — реальные ученые из сильнейших научных центров!


https://coollib.com/b/428217/read#t9
493 684834
>>84792

>С того, что пулька 22 калибра в состоянии убить человека.


Смотря куда попадёт.

>Если человек в скафандре, то для того чтобы продырявить скафандр большая мощь не нужна.


Если уж собрался воевать в космосе, то броню никто не отменял.
494 684836
Вообще, на тему оружия:
https://www.youtube.com/watch?v=F7qqHDViFko
495 684842
>>84816

>калибры больше тупа юзлесс


ПРОБИЛ ДЫРКУ В СКАФАНДРЕ 22lr
@
ПРОТИВНИК УСПЕЛ ДО ТЕБЯ ДОБЕЖАТЬ ДОПРЫГАТЬ И ПОРЕЗАТЬ НОЖОМ

ЭКИПИРОВАЛСЯ 22lr ДЛЯ КОСМИЧЕСКОЙ СТРЕЛЬБЫ
@
ПРОТИВНИК СДЕЛАЛ ПОДКЛАДКУ СКАФАНДРА ИЗ НЕСКОЛЬКИХ СЛОЁВ КЕВЛАРА
496 684844
>>84842

>ПРОБИЛ ДЫРКУ В СКАФАНДРЕ 22lr


>@


ПРОТИВНИК ЗАТКНУЛ ДЫРКУ ПАЛЬЦЕМ
16330196219991.jpg460 Кб, 1414x2000
497 684858
>>84792

>Если человек в скафандре, то для того чтобы продырявить скафандр большая мощь не нужна.


Даже с продырявленным скафандром противник может наделать делов. Например тупо пристрелит тебя в ответ, но уже не из 22-го. Умрешь раньше его.

>В итоге имеем имбу в виде автомата с большим магазином и лёгким боезапасом.


Этот патрон неудобен для больших магазинов из-за закраины. Редкое автоматическое оружие на его основе - несерьезные развлекательные пукалки, а требуется надежность автоматики, высокое останавливающее действие, а также потенциал для прикручивания механизма компенсации отдачи. Его главное достоинство - дешевизна, что в условиях космоса очевидно ничего не значит. Возми тогда другой, более совершенный патрон малого калибра.

Из существующего прямо сейчас в вакууме решает пикрил и его универсальность.
498 684869
>>84858

>Из существующего прямо сейчас в вакууме решает пикрил и его универсальность.


Джвачую, и противника пристрелить, и скорость для гравитационного манёвра набрать.
499 684886
Что произойдёт, если вдруг кто-то из экипажа МКС зайдёт на спейсач? Интересуют масштабы хлопка и уровень задымления. Сильно ли изменятся параметры орбиты станции? И вообще, возможно ли это, может, там на такой случай превентивные меры безопасности предприняты, файервол поднят и прочий роскомнадзор.
500 684887
Ох, как я люблю разговоры про оружие в космосе.
>>84786>>84792
>>84858
Господа-оружейники, какие есть мысли по поводу охлаждения огнестрельного оружия?
501 684892
>>84858

>Этот патрон неудобен для больших магазинов из-за закраины. Редкое автоматическое оружие на его основе - несерьезные развлекательные пукалки, а требуется надежность автоматики, высокое останавливающее действие, а также потенциал для прикручивания механизма компенсации отдачи. Его главное достоинство - дешевизна, что в условиях космоса очевидно ничего не значит. Возми тогда другой, более совершенный патрон малого калибра.


Ну можно любой аналог взять. Те же 5.7 мм или 4.6 мм.

Я не вижу смысла в винтовочных калибрах в космосе, так как в космосе любой патрон будет на вес золота. И мелкашку возить на станции куда проще, чем винтовочные калибры, так как мелкашки можно тупа привезти больше.

Насчет надежности — оружие под 22 калибр обладает достаточной надежностью. Ему не нужна слишком большая масса чтобы гасить отдачу, так что можно заняться этим >>84887

>Господа-оружейники, какие есть мысли по поводу охлаждения огнестрельного оружия?


вопросом, не снижая характеристик оружия ради того чтобы впихнуть нормальный калибр космосолдату/астросолдату, который в условиях космоса все равно будет обладать избыточной мощностью при дефиците количества патронов.
502 684893
>>84858
И да, очевидно, люди будут воевать друг с другом, а не с ксеноморфами.

>Из существующего прямо сейчас в вакууме решает пикрил и его универсальность.


Ну и сколько дробовых патронов можно доставить в космос по сравнению с мелкашкой? Пускай вот возьмем 410 калибр как самый компактный и относительно мощный. Он больше мелкашки по объему и по массе, и значит что ты не возьмешь много этого калибра. Это не упоминая, что оружие еще будет сложнее доработать чтобы космосолдата/астровоина не сносило нахуй отдачей.
503 684900
>>84892

>дефиците количества патронов


Спейсач в прямом эфире обосновал нужность холодного оружия.
504 684901
>>84900
А вообще даже без патроного голода.
Опасность пробить обшивку.
Опасность повредить критическое оборудование.
Мизерная дистанция практического боя что внутри станций и кораблей, что на обшивке.
Пуля дура, нож молодец.
505 684904
Да не нужен в космосе классический огнестрел. Будут ножи, мелкокалиберые безоткатные и реактивные орудия.
506 684932
>>84904
Нужен, иди нахуй
507 684947
Допустим, я высажусь на неизвестную мне планету в неизвестном положении относительно своей звезды, с неизвестными мне орбитой, траекторией и прочим. При этом, строго надо мной, прямо по вертикальной оси, перпендикулярной поверхности планеты, запускается пускай 5 ярких-преярких спутников светлячков, которые в ясную ночь видны, и даже в ясный день, первый на высоте 60км от поверхности со мной, второй 80, третий 100, четвёртый 120 и пятый 140.
Всё, что я смогу узнать, не имея иных способов измерения, это то, что мне скажут спутники. А что они мне скажут? Массу? Объём? Плотность планеты? Разницу во времени между полётами друга разве что
508 684950
>>84887
Для полуавтомата хватит радиатора побольше, заодно оружие будет выглядеть громоздко и футуристично, как в фильмах. Для автомата - жидкостное охлаждение испарением Максима, но нужно продумать, чтобы пары не загораживали обзор и не демаскировали.

>>84892

> в космосе любой патрон будет на вес золота


"И одна винтовка на двоих."
Применение оружия это очень редкое событие. Оружие, летавшее в космос (ижская трехстволка и макаровы), стреляло кажется всего один раз чтобы отпугнуть зверя после посадки или вообще не стреляло. Для любого ствола хватит 2-4 запасных магазинов, а там или ты пристрелишь врага, или он тебя.

>>84893

> Ну и сколько дробовых патронов можно доставить в космос


Достаточное количество для возможного боя. Мы вроде рассматриваем ситуацию что-то вроде конфликта на диком космическом западе, с существующим сейчас оружием? В большинстве таких ситуаций хватит трех патронов. Один подранить, второй добить. Третий - предупредительный в воздух вакуум, для отчетности. На один ствол хватит 30-40 патронов разного типа и навески.

А еще дробовик позволяет применять спецбоеприпасы: осветительный, слезоточивый, шприц с галоперидолом и др. В ситуации боя внутри модулей, чтобы не разгерметизировать их, пригодятся патроны с мелкой (утиной) дробью и минимальной навеской, либо нелетельные, с резиновой пулей. Мирные спецбоеприпасы могут понадобится гораздо чаще, чем боевые. Можно накрутить на ствол насадку и стрелять винтовочными гранатами инженерно-саперными боеприпасами по астероидам. Можно отстреливать ложные цели - пачку диполей, маркировать объекты на расстоянии флуоресцентной или светящеся краской, гарпунить спутники и вообще делать много чего полезного.
509 684953
Насколько быстро Земля движется в пространстве? Мы же каждую секунду меняем положение в пространстве вселенной. Пиздец. Если об этом думать достаточно долго - начинает укачивать
510 684955
>>84953

>Мы же каждую секунду меняем положение в пространстве вселенной.


Относительно чего?
511 684975
>>84950

> А еще дробовик позволяет применять спецбоеприпасы: осветительный, слезоточивый, шприц с галоперидолом и др. В ситуации боя внутри модулей, чтобы не разгерметизировать их, пригодятся патроны с мелкой (утиной) дробью и минимальной навеской, либо нелетельные, с резиновой пулей. Мирные спецбоеприпасы могут понадобится гораздо чаще, чем боевые. Можно накрутить на ствол насадку и стрелять винтовочными гранатами инженерно-саперными боеприпасами по астероидам. Можно отстреливать ложные цели - пачку диполей, маркировать объекты на расстоянии флуоресцентной или светящеся краской, гарпунить спутники и вообще делать много чего полезного.


Я сдаюсь, дробовик будет лучше. Двустволка какая-нибудь, универсальная пушка получается, без рофлов.
512 684994
>>84955
Ну относительно центра млечного пути например
image.png1,2 Мб, 1000x682
513 685019
>>84975
Да просто дух захватывает, сколько всего можно сделать с дробовиком. Будущий покоритель космоса это однозначно брутальный чэд в скафандре с дробовиком за спиной. А если примут какую-нибудь международную конвенцию о запрете разработки специализированного космического оружия, то он останется оружием выбора космонавта на последующие столетия.
514 685032
>>84953
Вокруг Солнца - 30 км/с, вокруг центра галактики - вроде бы 200 км/с
515 685033
>>85019
СТРЕЛЯЕШЬ ИЗ ДРОБОВИКА В ЧУВЫРЛУ ГИГАНТСКУЮ ДУРНОХАРАКТЕРНУЮ
@
ПОЧЕМУ Я НЕ ВЗЯЛ ВИНТОВКУ?
516 685037
>>85033
ПОТОМУ ЧТО ДРОБОВИК БЫЛ СО СВЕРЛОВКОЙ ЛАНКАСТЕРА
517 685076
>>85033
Пулей Полёва заряжать надо было
518 685096
>>85076
frag-12 с бронебойным наконечником, чтобы поглубже в НЕХ уходил. Вполне себе болтер.
Кстати, а шо там с отдачей в космосе?
image.png25 Кб, 1030x571
519 685098
Объясните один момент
Допустим есть сверхлегкий спутник
С одной из его сторон закреплен массивный магнит, который притягивает его

Он будет двигаться в открытом космосе или нет? Если нет то почему?
520 685109
>>85098
Космический корабль на принципе Мюнхгаузена? Патентуй немедленно!
521 685111
>>85109
Это я вам основную задумку не раскрыл, там просто пушка
522 685114
>>85111
Гауссова, надеюсь?
523 685115
>>85114
Нет. Куча мелких магнитиков будут менять полюса и шатать тело и задавать ему равномерное движение в открытом космосе.
524 685119
>>85115
А зачем полюса-то менять? Поотпиливать, например, южные, и налепить их на корабыль, а остальные выкинуть. Ну или на другой налепить, главное не пытаться в стыковку.
525 685145
>>85096

>шо там с отдачей в космосе?


Да чуть сильнее, чем в атмосфере. Это если без ДТК
526 685149

>срутся про беззадачный в вакууме огнестрел


>ни слова про термодинамику


>ни слова про холодную диффузию в вакууме


Без продуманной под вакуум конструкции, минимизации движущихся частей, и их смазки вакуумным маслом - хана вашим пукалкам, все детальки слипнутся к хуям. Даже гильзы прилипнут куда-нибудь обязательно, к магазину хотя бы.
Пластик будет газовать и разлагаться под УФ.
В невесомости сильно не постреляешь - отдача кувырнёт, потому что нащупать ЦТ нечем.
На Луняшке сильно тоже не постреляешь - сцепление с почвой никакое, скользишь почти как по льду, любая отдача тебя опрокинет. Безоткатка нужна тогда уж.
Охлаждать эту хуйню вообще нечем.
527 685151
>>85149
РПГ с газоотводом без отдачи и нагрева
528 685160
Дальность радарного обнаружения в космосе: от чего зависит, какие-то конкретные цифры?
Просто не знаю даже как грамотно сформулировать вопрос. Кажется очевидным ответ, что дальность обнаружения в принципе не ограничена, и вопрос только в мощности излучения и чуткости приемной антенны, раз уж фотографии планет у других звезд делают. Хочется просто какой-то конкретики, например, какого размера объект обнаружит какой радар на каком расстоянии, и наоборот, какого размера объект и на каком расстоянии такой же радар уже не сможет засечь. В гугл ходил, там все данные по обнаружению космических объектов радарами на поверхности Земли, или наоборот, обнаружение объектов на поверхности орбитальными радарами, а меня интересует система космос-космос. Будет наверное яснее, если сразу скажу, с какой целью интересуюсь - с целью проработки возможностей противоракетной обороны (да, я с соседней доски). Типа на каком расстоянии радар сможет засечь приближающуюся ракету, или корабль, который пускает ракеты.
529 685173
>>85149
Срётся только слепошарый гринтекстовый петух.
Беззадачность огнестрела - твой лично высер.
Вопросы охлаждения рассматривали выше.
Холодная диффузия - миф.
530 685180
У меня такой вопрос.
Возьмем космическое тело без атмосферы. Или со слабой атмосферой, например луна.

Делим луну на половину. И на этой половине устанавливаем ракетные двигатели. Обычного размера, очень много.

В один момент - одновременно запускаем, пока не выгорит все топливо.
Реально ли так дать импульс космическому телу и сбить его орбиту?
532 685189
>>85180
Ну смотри. Положение центра масс у тебя остаётся неизменным. Вылетающие из сопла молекулы подчиняются распределению Максвелла. Те, что помедленнее, не дотянут до скорости убегания и ёбнутся обратно на Луну, образовав жиденькую атмосферу. Быстрые же улетят в ебеня. Вот они-то и придадут твоей луне импульс, изменив её орбиту.
533 685190
>>85189
То есть это не окажет никакого заметного влияния
534 685192
>>85190
Странный качественный вывод при отсутствии каких-бы то ни было количественных оценок.
535 685198
>>85192
Нет, это я пытаюсь понять какой ты вывод из этого сделал
536 685207
Не могу найти где обсуждают авиасимуляторы?
1630915033886.gif220 Кб, 500x500
537 685228
Пришло время перекатываться >>685225 (OP)
Пришло время перекатываться >>685225 (OP)
Пришло время перекатываться >>685225 (OP)
538 685250
>>85207
В /vg/ был тред.
539 685251
>>85173

>Беззадачность огнестрела - твой лично высер.


Держи в курсе, воеван.

>Вопросы охлаждения рассматривали выше.


Даже близко не рассматривали, промямлили что-то.

>Холодная диффузия - миф.


А вот тут ты идёшь нахуй.
540 685331
>>79708 (OP)
Как у лоха педального в этой сфере у меня есть вопрос к истинным знатокам. Как устроиться астрономом в обсерваторию? Обязательна ли например, докторская степень, или туда берут и просто с бакалавром или магистром?
541 685332
>>79708 (OP)
Вопрос к гиперпространству. Если мы не можем представить иные измерения, но можем математически их "показать", то почему бы например не взять нейтрино, который видёт себя так, будто он как раз находится в тех "иных" измерениях, и посчитать теми же методами которыми объяснялись измерения то, каким образом нейтрино проходит сквозь объект? По идее тогда можно попробовать визуализировать измерения, в которых он также находится. Ну, это если конечно, гиперпространство и правда существует. Наверное, при наложении на квантовую физику это всё становится нереальным, чёрт знает... (Звучит бредово, жду пока меня кто - нибудь разнесёт!)
542 685364
>>85332

>нейтрино, который видёт себя так, будто он как раз находится в тех "иных" измерениях


С чего бы это оно в иных измерениях? Оно просто маленькое пиздец, лёгкое и не имеет электрического заряда, что позволяет ему летать сквозь эти ваши атомы, ни на что не отвлекаясь.
543 685382
>>85364
А, окей. Просто ощущение будто где то читал именно про то, что нейтрино могут иметь такую способность как раз из за связи с большим количеством измерений чем мы видим
544 685389
>>85149

>Без продуманной под вакуум конструкции, минимизации движущихся частей, и их смазки вакуумным маслом - хана вашим пукалкам, все детальки слипнутся к хуям. Даже гильзы прилипнут куда-нибудь обязательно, к магазину хотя бы.


Ну ахует капитан очевидность пожаловал, здрасти.

>Пластик будет газовать и разлагаться под УФ.


А трава зеленая, дебил. Хранить патроны в патронташе не?

>В невесомости сильно не постреляешь - отдача кувырнёт, потому что нащупать ЦТ нечем.


Ты дельту уже посчитал? Такого рахита как ты конечно кувыркнет, особенно если нет мозгов стрелять, зависнув в вакууме.

>На Луняшке сильно тоже не постреляешь - сцепление с почвой никакое, скользишь почти как по льду, любая отдача тебя опрокинет.


Пиздец ты даун. Скользи вон отсюда.

>Охлаждать эту хуйню вообще нечем.


Пиздец ты совсем конченый даун.

И да, расскажи нам про свое "вакуумное масло", дебил ты вакуумный.
545 685422
>>85331
У нас в стране нет бакалавриата по астрономии, только специалитет. Хотя можно в некоторых вузах будучи физиком специализироваться на вопросах космоса
546 685498
>>85422
Я и не из России. Не знаешь насчёт этого, пустят ли в какую нибудь европейскую обсерваторию просто с образованием бакалавра?
547 685517
>>85498
Понятия не имею тогда, от научных работ зависит во многом
IMG0462.JPG2,7 Мб, 5152x3864
548 689345
>>79708 (OP)
Здрасте! Возник тупой вопрос, поэтому обращаюсь сюда. Есть фотография, а на ней что то похожее на планету, в правом нижнем углу. Если это так, а не тупо засвет какой нибудь, то не мог бы кто то сказать или предположить что это за фрукт?
Тред утонул или удален.
Это копия, сохраненная 17 июня 2022 года.

Скачать тред: только с превью, с превью и прикрепленными файлами.
Второй вариант может долго скачиваться. Файлы будут только в живых или недавно утонувших тредах. Подробнее

Если вам полезен архив М.Двача, пожертвуйте на оплату сервера.
« /spc/В начало тредаВеб-версияНастройки
/a//b//mu//s//vg/Все доски